Does math have big scary teeth or something?
October 17, 2009 9:30 AM   Subscribe

Why do people hate math and science so much?

I study cosmology for a living. Whenever I tell people this (and I usually use the word astronomy), I usually get one of three responses: 1) 'Oh, well... that must be hard' 2) 'I couldn't even begin to understand any of that' or 3) 'Cosmotology! Can you do my hair? Haha!'

This kind of gets to me after a while, and I was thinking last night about how people generally dismiss math and science (my passions in life!) from conversation. To the best of my consideration there are two reasons for this, certain people are actually incapable of understanding math and science, or they are afraid of it. I really hope the answer is the latter.

I then began to think about people's ideas of a well-rounded, well-educated individual: he/she is expected to have read Shakespeare, but he/she is not expected to know the two postulates of relativity, or all three of Newton's laws, or what a derivative is. This is at least my impression from my experiences in my tier-1 college and grad school (you can probably guess where from my username).

So what's the deal?

Why do elementary school teachers teach multiplication tables for 6 years and call that math?
Why do people tell women that they can't do math?
Why do people treat me, a scientist, as if my work is that much more esoteric and specialized than someone else with a similar degree in another field?
Is the ability to do math a learned behaviour, or is it somehow intrinsic knowledge?

Basically, what the hell is so scary about math?!
posted by chicago2penn to Science & Nature (124 answers total) 101 users marked this as a favorite
 
Obnoxious, narrow-minded, impatient math teachers is what. Most people get burned and scarred for life.
posted by Namlit at 9:39 AM on October 17, 2009 [37 favorites]


If someone has exposure to a subject only via school, they'll tend to have negative feelings associated with it due to how poorly things are taught.
posted by lsemel at 9:42 AM on October 17, 2009 [3 favorites]


Because it is abstract. Because it is dry. Because it makes your brain hurt.
And although math can be beautiful, and some people love it, math contains no love or emotion, as do art or music or literature.
posted by weapons-grade pandemonium at 9:45 AM on October 17, 2009 [7 favorites]


I find math very frustrating, because no one ever explained it to me. Every math class I had said "Do this equation this way. Get this answer. Move onto the next question." Nobody said why it worked, or what it was for, or what it meant.

It wasn't until I was in my thirties that I realized (entirely on my own) that:

1+1=2

Isn't simply a addition problem for which I should know the rote answer. It's also an elegant expression, this combination of ones means the same thing as this two. They are the same word, written in two different ways.

So when all those teachers taught me the process, but not the meaning- I never understood it. I never understood what I was doing; consequently, I failed. If I forgot my multiplication table, I was lost with only process to guide me. And asking what it meant, or how to use it was considered smart assery in the extreme.

So even though I *love* the sciences, I never had a chance to take higher courses in them because I didn't understand the math.

I can't speak for everybody. But that's why I found math scary and upsetting. I didn't understand what it was for, or how it worked, so I did very poorly in it. You either get the answer right in math or you don't, and more often than not, I didn't. So I gave up on the dream of being an archaeologist because I "wasn't good at math."
posted by headspace at 9:45 AM on October 17, 2009 [37 favorites]


I don't think your assumptions are exactly right. I don't think people are "expected to have read Shakespeare" any more than they are expected to understand differential equations. Also, plenty of people I know love both subjects.

That being said, assuming you are American (as am I), I know that where I grew up there was a profound anti-intellectual component to the socializing I was subject to in high school. Based on conversations I've had with other Americans, this was true for them also. It's not specific to math and science, but all intellectual endeavors. Perhaps that's a partial answer to your (in my opinion somewhat poorly conceived) question.

Why do elementary school teachers teach multiplication tables for 6 years and call that math?

That's not what I was taught; was that what you were taught?

Why do people tell women that they can't do math?

This is another big generalization that could be phrased better, but I recognize that there is a problem with getting women into the mathematics and sciences. Institutionalized stereotypical assumptions about gender roles is the quick and dirty answer.

Why do people treat me, a scientist, as if my work is that much more esoteric and specialized than someone else with a similar degree in another field?

Again, are you sure this is unique to scientists?

Is the ability to do math a learned behaviour, or is it somehow intrinsic knowledge?

Certainly learned, but it would be wrong to say some people don't have some better skills built-in.
posted by dubitable at 9:45 AM on October 17, 2009 [1 favorite]


Math and science in schools are taught in a very results-oriented way: it's about finding or knowing the right answer, not about ways of viewing the world or processes for finding and testing ideas. This means that people associate math and science with being told they are wrong (and therefore "I'm bad at math" or "science is hard").
posted by mbrubeck at 9:46 AM on October 17, 2009 [6 favorites]


And although math can be beautiful, and some people love it, math contains no love or emotion, as do art or music or literature.

Hmm...I don't know if that is true. I think this is really a matter of your experiences with those subjects.
posted by dubitable at 9:46 AM on October 17, 2009 [1 favorite]


It's not scary, and it's not that people are incapable of understanding. It's that maths and science ARE specialised, esoteric knowledge, in the same way as, for example, how to make sushi is, or HTML, or colour theory, or who wrote which Beatles song, or how to conjugate German verbs. I think you underestimate people, and overestimating the centrality and importance of your own interests to people's lives.
posted by dydecker at 9:48 AM on October 17, 2009 [8 favorites]


Yeah, I get that some people find the concision and compactness of mathematical expression to be beautiful but unless this beauty can be made germane to my daily life (for example the presence of the golden ratio in nature) a lot of it seems abstract and unimportant.

I understand that it's not unimportant but at the same time it is hard to understand why it is relevant to my life.
posted by dfriedman at 9:48 AM on October 17, 2009


I think that most people who really love math and science go into careers other than K-12 teaching. There are just a lot of opportunities for those people to have better-paid, higher-status jobs. That's even more true now than it used to be, because in the past women who were good at math and science were pushed into K-12 teaching. Now they can be scientists and engineers, too, so there's not a huge pool of really great, committed, talented K-12 math and science teachers. On the other hand, if you really love English or history, K-12 teaching is still one of the more attractive professions. For that reason, kids are more likely to have inspiring K-12 English and history teachers than inspiring K-12 math and science teachers.
posted by craichead at 9:50 AM on October 17, 2009 [6 favorites]


ChatFilter? It may be your confirmation bias, chicago2penn. I am also a scientist (molecular virologist), and this hasn't been my experience at all with the people I meet outside of work.
posted by halogen at 9:51 AM on October 17, 2009 [1 favorite]


Uh, I studied history in college and 75% of people I tell that say, "Ohh, I hated history," or "I suck at history." On the other hand, people who study math/science/engineering/etc. seem to get that response less often. They usually get more of an, "Oh wow, that's impressive," type answer.

Also, Shakespeare wrote plays. Plays are a form of entertainment. Books can also be a form of entertainment. A large percentage of people read or watch drama (including TV and movies) for fun. Even if they don't pursue these things academically, they are entertaining for those people. The closest most people will ever get to doing math for fun is sudoku.

I love science and appreciate math, but am not very good at them. I might read articles on scientific topics or watch Nova, but I'll never be able to have a real conversation about it with a real scientist or mathematician. I think a lot of people are the same way, and it's intimidating to try to talk about those things in any more than an armchair way.
posted by ishotjr at 9:53 AM on October 17, 2009 [2 favorites]


Why do elementary school teachers teach multiplication tables for 6 years and call that math?

Not all do. My kids' public school district doesn't. I remember my first graders doing basic fractions. My fourth grader is doing harder math than I did in middle school. My 7th grader is in Algebra 1.

Why do people tell women that they can't do math?


I will agree that that used to be more of a problem than I think it is today. In my circle of friends I have an investment banker, a retirement funds portfolio manager, a micro biologist, and and archaeologist - all women. That's along with artists, moms, teachers, etc. I'm in my mid-thirties.

Why do people treat me, a scientist, as if my work is that much more esoteric and specialized than someone else with a similar degree in another field?

I don't know, but I suspect from the reaction you described when you tell people what you do that you aren't hanging out with open-minded, like-minded people. Cosmology is way cool and while it's not my passion, I respect that it's yours and I love hearing about it. And I can tell you for certain that you'd be more than welcome to hang out with me and my friends and talk about science.

Is the ability to do math a learned behaviour, or is it somehow intrinsic knowledge?

I suspect it's a little bit of both. I find it harder to do math than, say, understand Shakespeare, but I can do it if I give it time and thought.
posted by cooker girl at 9:53 AM on October 17, 2009


math contains no love or emotion

Hmm...I don't know if that is true.


It seems self-evident to me. However, it also contains no rage, envy, jealousy and hate.
posted by Obscure Reference at 9:54 AM on October 17, 2009 [2 favorites]


Namlit nailed it. Except that it certainly doesn't end with primary or high school. My worst math teacher ever was my Calc II prof at university... he was a bright mathematician, but wasn't at all conscious of the process he was going through, and so, when asked a question, he'd simply point at the same incomprehensible series of steps on the board and shout, "Like this! Like this!"

In the US, we've decided that math is identical to arithmetic. And so that's what we teach. And, frankly, advanced arithmetic is hard. An integration by parts? Domain transforms? All things I understand and can easily get a computer to solve numerically, but which I find tedious and difficult to do by hand. And since the emphasis is on arithmetic competence, not mathematical reasoning, teachers refuse to let students use calculators or computers to really explore mathematics.

It's my understanding that the math == arithmetic thing started after the failure of the so-called "new math" initiative during the baby boom generation. Teachers, who'd been teaching the 3 R's, didn't know what the fuck they were doing; parents, who were expected to help with homework, didn't know what the fuck they were doing; and students, as a result, didn't know what the fuck they were doing.

When most of your life's experience with math has been to work tediously at it for hours only to be told you're wrong, with no help or explanation available as to why it's wrong... well, it leaves a bad taste in your mouth.

What's more, math is a field in which you can get an objectively wrong answer. In a world where everybody's on the honor's list, A's are "average", and every nursery-school grammar essay is given points for effort, the cold, harsh light of an answer actually being incorrect in some sort of unarguable way is really frightening for some people. And since so few can ever seem to teach math very well, even at the college level, very few of those students ever get to experience the joy you and I do when something is unarguably, objectively right.
posted by Netzapper at 9:54 AM on October 17, 2009 [17 favorites]


That is an open-ended question, but I do thing, in my experience, as others have said, that how mathematics has been taught in the US sucks. I think that poor teaching is actually encourged by many math professors so that there is a "priesthood" mystique.

Mathematical notation isn't easy for the average person to pick up. I understand its absolute utility, but like musical notation, it is a specialized notation. I love science, and always wished I had gotten math but it wasn't until I was an adult and read math books for the general reader, like Berlinski's "A tour of the calculus," that I learned what the hell the point of it all was. THen I decided that I will have to learn the language of math the way I would have to learn a foreign language. Not easy and I met with varying success.

I am not sure how math should be taught but I think the general whys and wherefores of calculus, how it measures change and limits etc., and how that is an amazing thing, all w/o math notation and then, only then, when you really grok it, you can use the notation as a convenient tool.

Second reason I think is that math, like science is a useful tool, but one that can be used for nefarious purposes. The good uses are sort of under the radar, ubiquitous though it is, but the bad uses, whether "quants" who helped to ruin the economy, or mendacious politicians who use skewed "statistics", the perception of math isn't good. That is sad.

Perhaps another good course in high school college, which would be required is something like "survival math," or "survival statistics - how to spot a cheat, a liar, a thief."

Math doesn't have big scary teeth. Math teachers often do. Really. I've asked a lot of stupid questins in my student days. I've never got a withering eye-rolling "Jeez what an idiot look" from any teacher - except a math teacher.

Would that there were a movie called "Dead Mathematicians society" that had a rogue teacher celebrate Cantor, Riemann, Gauss, and Godel with his students. (So long as Robin Williams wasn't in it.)
posted by xetere at 9:55 AM on October 17, 2009 [4 favorites]


In my experience, it's easier for people in the humanities and social sciences to make their work understood to people in outside fields than it is for people in advanced math or physics. I never remember what Derrida wrote or why it mattered, but I can have a reasonably intelligent conversation with my friend who is a literature PhD student nonetheless. Having a similarly intelligent conversation with my friend who does theoretical physics, on the other hand, seems impossible to me because even the most basic description of his work is so complicated and alien compared to anything I've ever done or studied.

Maybe you could take it upon yourself to find a way to communicate what you do more effectively to people who feel insecure in their ability to understand math and science.
posted by Meg_Murry at 9:55 AM on October 17, 2009 [2 favorites]


math contains no love or emotion

Hmm...I don't know if that is true.

It seems self-evident to me. However, it also contains no rage, envy, jealousy and hate.


I guess this is getting a bit chatty, but: I suppose what I meant really was how people approach it, and people certain approach math with a lot of love and emotion. But reading back, I think I was misunderstanding the point--it would be hard for me to disagree with you in that math does not itself contain love and emotion.
posted by dubitable at 9:59 AM on October 17, 2009


Why do elementary school teachers teach multiplication tables for 6 years and call that math?
Because it's necessary to understand multiplication before a person can understand more complicated math? Maybe I'm missing something in that question.

Why do people tell women that they can't do math?
Maybe some people do that, but I'm a highschool student, and there are just as many girls and boys in my BC Calc class.

Why do people treat me, a scientist, as if my work is that much more esoteric and specialized than someone else with a similar degree in another field?
Because math and science is harder to understand than a similar degree in history or literature. People understand history and literature, and if they got plunked into a high level course discussing one of those topics, they probably would be able to understand most of what was being said. On the other hand, if someone was plunked into an advance math class they would likely have no idea of what was going on.

Is the ability to do math a learned behaviour, or is it somehow intrinsic
knowledge?

I'd definitely say its learned behavior. Some people have a harder time understanding it than others, but everyone can learn to understand it. There are very few people that just automatically know how to do math.
posted by kylej at 10:09 AM on October 17, 2009


I think the reason that so many people hate math is because it can be very unforgiving. When coupled with sub-par teachers, the unforgiving nature of math can be extremely discouraging. If you're writing an essay, a misspelled word or a poorly constructed sentence in the first paragraph doesn't ruin the whole paper. Granted, if you're mistaken about a broad general idea, then that might be a problem, but a tiny mistake is completely irrelevant in the face of the larger work. If you make an early mistake in a math problem, however, all of your work can become incomprehensible. In order to discover what went wrong, you have to start over and carefully comb over the problem to find out where you reversed a sign, or mistook one value for another, or made a simple arithmetical error.

Science relies so much on math that I think it draws fire by extension. People who don't like math but find scientific concepts interesting still aren't often willing to struggle through the math required to actually measure things.
posted by solipsophistocracy at 10:12 AM on October 17, 2009 [2 favorites]


Bob need to buy beer for a party which will be attended by 35 people. If he wants to have at least 2 beers per person, how many six-packs should he buy?

I find that many people who would have no trouble being Bob and buying the right amount of refreshments would suddenly panic if asked the same question on a math exam. Framing it as a test of their knowledge makes it scary, and the subsequent anxiety (unless they always carry around an emergency Xanax) makes the solution difficult. For many who can't do mathematics, the problem is more one of anxiety than of knowledge or ability.

Several previous comments blamed how they had been taught, but I've found (I'm currently teaching a statistics course) that most students don't want the material to be made understandable. They want recipes to memorize. If I try to give insight into why a formula is the way it is, they don't find it helpful, nor do they wish to see more than one way of solving a problem, forcing them to have to think about which one to use and/or study. These students are non-science majors in a required course, thus not a (statistically) random sample.
posted by Obscure Reference at 10:19 AM on October 17, 2009 [4 favorites]


I dislike math because there is no wiggle room.
I dislike math because of the terms "signed numbers" and "integers." Who the hell thought these up, and why?
I dislike math because I resent the fact that it used to be called arithmetic, but now it's called "Math"; short and nasty.
And most of all, I hate hate hate the phrase, "You do the math!"
posted by BostonTerrier at 10:19 AM on October 17, 2009


1) The quality of maths/physics education below university level. In particular I found it often teaches little bits of everything and you never get the beauty and consistancy of studying something from the axioms up.

2) well-educated individual: he/she is expected to have read Shakespeare, but he/she is not expected to know the two postulates of relativity

This thought has been around for quite a while; look up C.P. Snow's "The Two Cultures" lecture/book.

3) I think this whole thing is amplified in Western culture because we tend to view mathematical ability as a talent rather than an aspect of hard work, as Malcom Gladwell discusses in this interview (maybe in part 2).

Also because dfriedman's point.

But given all this I'm fairly sure that if I met someone who was an artist I would say "wow, I really can't draw" and they would probably find it just as annoying in the same way.
posted by Erberus at 10:22 AM on October 17, 2009 [1 favorite]


Several previous comments blamed how they had been taught, but I've found (I'm currently teaching a statistics course) that most students don't want the material to be made understandable. They want recipes to memorize.
I've found that, too. But I've also found it to be true with humanities courses. I'm a college advisor, and I actually get a lot more students complaining about how humanities professors demand that they think than about how math and science professors do. I think that a lot of students come out of high school with an expectation that they'll be given answers to memorize, and they have trouble across the board in college when there's an expectation that they'll understand concepts and apply them to unfamiliar questions or problems.

I've seen a fair number of smart students really blossom in classes that demand that they understand concepts, though. That's true both of humanities classes and "science for poets" type classes. A lot of kids think they hate history, but really what they hate is memorizing dates. Similarly, a lot of students who think they hate science realize that they really love classes that focus on scientific concepts, rather than memorizing formulas and plugging numbers into them.
posted by craichead at 10:29 AM on October 17, 2009 [2 favorites]


I disagree that there is no beauty in math. There is. But the way children are taught math is generally very rigid; for the percentage of kids who struggle to learn in a given manner, math becomes something you get through, not something you love.

I was very fortunate in my education that I had teachers who would help me learn math visually, from my earliest days taking away blocks and re-counting them to understand subtraction to the teachers who would take me outside and help me learn to calculate the distance from a 3rd story window to a signpost to the professor who at least gave me the take-away of an abiding love for the beauty of spreadsheets when I was dying on my ass in Statistics class. While I am still not good at math, I don't hate it, nor do I consider people who are good at it to be some kind of strange and supreme gods.
posted by DarlingBri at 10:29 AM on October 17, 2009


From what you're saying, chicago2penn, people's reactions to your work (the lame cosmetology joke excluded) sound more like they're in awe of your work than dismissive of it. They think it's hard because for them, it is. They say that they couldn't understand it because probably, they couldn't. And that doesn't mean they're dismissing what you do, or math and science in general. It means at some point in their life, they decided they liked other things more than math and science and did something else with their life.

For example, I can do math. I took calculus in high school and found it interesting at times but mostly frustrating. I thought the concepts were interesting, but I didn't enjoy doing math homework, I didn't enjoy the way of thinking math required, nor did it come to me easily. I find myself now wishing it did come more easily to me, as my interests are forcing me into a more math-y path than I would have expected in high school. I'm getting through the math because it's related to things I do like. I still don't like math. It isn't easy for me. And so, I am choosing to do other things with my life and my time at college. I know quite a few math/science people here who hate English and history (and I go to a liberal arts college that tends to attract people who like these things and requires that people take classes in a variety of subject areas).

So this is getting long-winded, but basically, I don't think that people are incapable of math and science, nor are they afraid of it necessarily. I think a lot of people find math tedious and difficult, just as many people (perhaps fewer) find writing English papers tedious and difficult. Thus, they choose to do other things with their life. I don't think that makes people dismissive of math and science, I think it makes them admire the intellect of people who can do what they can't (or won't).
posted by MadamM at 10:37 AM on October 17, 2009 [1 favorite]


Regarding the responses you get when telling people your profession: I don't think most of them actually hate math and science, I think this is just some THING people DO. They have to form some kind of response to your statement but they know nothing about astrology/cosmology. It is in the public consciousness that these things are hard (for reasons others in this thread have addressed), therefore they say, "Oh, well... that must be hard" or "I couldn't even begin to understand any of that." This is a standard response to... anything you know nothing about. It's automatic and generally acceptable (even though annoying to you. and me.)

I study a fancy sounding science in school and I get those same responses. When asked my major, I found myself inadvertently taking on an apologetic tone and wincing facial expression. After I realized that, now I just say, "well, I could never be a (fill in the blank), I suck at (fill in the blank), and we need those too."
posted by bobobox at 10:39 AM on October 17, 2009 [3 favorites]


Math doesn't have big scary teeth. Math teachers often do.

Quite right. I blame elementary school for this kind of thing, and I don't care if it is because of baggage. I have always been slightly dyscalculic, and although I now do perfectly well by understanding my own issues, repeating calculations and using calculators wherever possible, I believe that elementary school teachers basically drove me away from math. Because I did so well in my other subjects, the reason I did poorly in math just had to be because I wasn't trying hard enough. I am jealous of scientists every day, because of their power and their passion for their work, but high-school stoichiometry defeated me and that was that.

This kind of thing embitters otherwise thoughtful humanities graduates, you see. People often hate what they can't get power over. If you find that you're misunderstood or hand-waved away, it may make you feel better to know that it's because you're incredibly lucky in your talent and training.
posted by Countess Elena at 10:39 AM on October 17, 2009 [1 favorite]


One article I like is this one by a mathematician on the subject (the meat of his argument starts on page 3).

He takes a pretty snarky stance sometimes, but his basic claim is that the current ways math is taught (making kids memorize formulas and regurgite them rather than focusing on meaning and discovery) are making kids hate math. Some of my favorites lines:
Mathematics is the art of explanation.

The saddest part of all this "math reform" are the attempts to "make math interesting" and "relevant to kids’ lives." You don’t need to make math interesting— it’s already more interesting than we can handle! And the glory of it is its complete irrelevance to our lives. That’s why it’s so fun!

Mathematics is viewed by the culture as some sort of tool for science and technology. Everyone knows that poetry and music are for pure enjoyment and for uplifting and ennobling the human spirit (hence their virtual elimination from the public school curriculum) but no, math is important.

At no time are students let in on the secret that mathematics, like any literature, is created by human beings for their own amusement; that works of mathematics are subject to critical appraisal; that one can have and develop mathematical taste. A piece of mathematics is like a poem, and we can ask if it satisfies our aesthetic criteria: Is this argument sound? Does it make sense? Is it simple and elegant? Does it get me closer to the heart of the matter?

Mathematics is the music of reason.
posted by Zephyrial at 10:43 AM on October 17, 2009 [27 favorites]


People learn to hate math by the common teaching method. They are given class deadlines and then poor grades, and later moved forward without mastery. This slowly causes anxiety. Anxiety equates to fear and loathing.

A solution would be cooperative education, where peers tutor peers, and it is important to note that both peers are learning math during this process. The teacher simply determines whether the student has achieved mastery.
posted by Brian B. at 10:44 AM on October 17, 2009 [2 favorites]


Some additional factors:

*For whatever reason mathematics seems to lend itself to, well, prodigies of different degrees. There are some people who generally find math generally deeply intuitive and, at least through calculus, not particularly difficult. If you're one of those people, I think you might have real problems understanding how difficult some people can find math just because they don't happen to be a minor (or major) prodigy like you are and don't find it immediately intuitive the way you do.

*In college, you have your choice of dull science-for-idiots classes that focus on rote memorization, or weedout courses such that if you were trying to make people believe that math (and science) are too difficult for the average college student, you would be hard pressed to improve on what exists.
posted by ROU_Xenophobe at 10:46 AM on October 17, 2009


Math and science in schools are taught in a very results-oriented way: it's about finding or knowing the right answer, not about ways of viewing the world or processes for finding and testing ideas. This means that people associate math and science with being told they are wrong (and therefore "I'm bad at math" or "science is hard").

I think this explains a lot of it among school kids. Being wrong in math or science classes is generally less unambiguous and more damaging to self-esteem than in other classes. This partly a consequence of how these subjects are taught, but partly intrinsic to the material.

As for the responses you described, I think Neal Stephenson addressed this in an op-ed about Star Wars a few years ago and also a bit in Anathem (and I think in some talks I've seen him give on Youtube, though I can't find them now). His basic point, if I understand it correctly, is that negative stereotypes of geeks stem from society's resentment at being dependent on technology they don't understand. In Star Wars, this is seen in the fact that Jedi intuition/religion is trumps technology, even though the Jedi understand technology too... in Star Trek, its seen in the Kirk/Spock dynamic. In one entertaining riff in Anathem (a sci-fi book that takes place on a world similar to ours), an instructor in a cloistered community of geeks provides a humorously long list of mostly negative stereotypes that outsiders have about their community members, all of which are seen in our own popular culture regularly.
posted by gsteff at 10:48 AM on October 17, 2009 [1 favorite]


er, Being wrong in math or science classes is generally more unambiguous
posted by gsteff at 10:53 AM on October 17, 2009


I hate math and science. I don't understand them, and they feel completely inaccessible to me. Here's my stab at explaining all the reasons why:

1. Schools (at least the ones I attended) teach science and math to the tests. There didn't seem to be passion in the subjects even for my teachers; it was all about memorizing facts that appeared arbitrary to me. No sense of wonder or puzzle-and-solve. Not even an organized system of logic that I could discern, grasp, and then rely on.

2. It's binary. Either you're right or you're wrong. I'm sure this isn't the case when you get to the more advanced, theoretical levels of either math or science, but it takes many years of that binary experience before you're ready to debate theory. So neither subject felt creative in the least.

3. I could never tell how to change my study techniques in order to improve. Math and science were completely counter-intuitive to me, and I could never "get it". The only answer I ever got when I asked teachers and adults how to approach learning it was practice. Well, if you do the same thing over and over, you'll get the same result over and over. If that result is failure, that's not fun.

Plus, someone once decided I was no good at math or science, and made the mistake of telling me that. From then on out, I'm sure part of it was confirmation bias. I have no idea who it was, or how early, but judging by the tip chart I keep in my wallet, it was way back in the basics years.
posted by nadise at 10:54 AM on October 17, 2009 [1 favorite]


It's that maths and science ARE specialised, esoteric knowledge, in the same way as, for example, how to make sushi is, or HTML, or colour theory, or who wrote which Beatles song, or how to conjugate German verbs.

I couldn't disagree more with this statement. The scientific method is central to so much of our lives. When you have people roaming around who don't understand that there's a difference between homeopathy and medicine, or think that flu vaccines will given them autism, that's a huge problem. At the core of this is a lack of understanding of how to think critically and test hypotheses.

And math, really? You're going to argue that society wouldn't be better served if people had a better grasp of statistics? Our risks of dying from a car accident or a heart attack are many orders of magnitude greater than of dying from a shark attack or terrorist bomb. Yet, we spend billions of dollars and many sleepless nights worrying about the latter two, while largely ignoring the former.

Global warming, H1N1, the energy crisis, health care - these are all areas of public debate that are dragged down into meaningless drivel because people lack a basic understanding of the core issues. Improved math and science literacy is one of our society's most pressing needs.


To the OP: You may want to examine your approach. The problem may be that you're not presenting your work in such a way as to make it interesting to the listener. Many scientists are bad at this, because it doesn't come naturally. You need to have a thirty second elevator pitch that concisely sums up what you do and why it's cool. You also need to have a five minute spiel that goes a little more in depth without losing a lay-person.

If your audience is bored, perhaps it's because you're boring.
posted by chrisamiller at 10:55 AM on October 17, 2009 [4 favorites]


Math doesn't have big scary teeth. Math teachers often do.

There are a lot of bad teachers out there--I was terrorized by history and English teachers. Perhaps subject matter with humans in it results more in teachers with empathy.

I also find that math has a lot of underlying ambiguities that people who aren't seeing them for the first time take for granted. 1/2: is it a concept? A number? An act of division?
How about that unary minus? It's glossed over when teaching the order of operations.
When 2 numbers are next to each other, are they being multiplied? What if one has a unary minus? What about two an a half, written as a 2 next to 1/2? The assumed to be logical notation is often as confusing as the English language for a non-speaker.

I tried to explain irrational numbers to my class by telling them how the Greeks were upset to discover that there could be 2 geometric lengths with no unit, however small, that could fit an exact number of times into both of them. I still don't know if they "got" it. People don't get upset about that kind of thing anymore.
posted by Obscure Reference at 10:55 AM on October 17, 2009 [1 favorite]


So when all those teachers taught me the process, but not the meaning- I never understood it.

This.

In my experience both as a horrible-at-math student, and then as a proselytizing convert who's ended up teaching basic math, the breakdown seems to begin at fractions.
posted by small_ruminant at 11:05 AM on October 17, 2009


At the core of this is a lack of understanding of how to think critically and test hypotheses.

No, in my experience, the core of the problems you cite is a lack of transparency on the part of the FDA and medical and pharmaceutical industries.
posted by small_ruminant at 11:08 AM on October 17, 2009


"2. It's binary. Either you're right or you're wrong. I'm sure this isn't the case when you get to the more advanced, theoretical levels of either math or science, but it takes many years of that binary experience before you're ready to debate theory. So neither subject felt creative in the least."

I'd like to point out that this is one of the good things about math, not a reason to dislike it. Math (and the basic hard sciences in general) is not something that can be debated, not something that has "two sides" that need explaining, not something that you can make up a bunch of woo-woo about to handwave away when you're not willing to think with precision. Certainly there are going to be various personality types that are turned off by their inability to BS their way through an education with a basis in fact, but, frankly, people who reject facts and logic aren't going to benefit usefully from pretending math and science are fuzzy, debatable, or benefit from self-indulgent stabs at "creativity."

Many other criticisms of math education and popular understanding of the sciences are valid and interesting perspectives. This one in particular just perpetuates a bullshit view of the universe that keeps humans ignorant and pliant.
posted by majick at 11:09 AM on October 17, 2009 [3 favorites]


Best answer: I don't think the main cause has anything to do with Math or Science per se. Someone upthread said that there's a profound anti-intellectual trend in America. I agree, but I think it specifically takes the form of A DISDAIN FOR PROBLEM-SOLVING SKILLS IN ACADEMIC SUBJECTS. This is just as true when it comes to Shakespeare as when it comes to Math.

The life-goal in America seems to be to get a well-paying job in which you don't need to think very much. I doubt this is a conscious goal, and it sounds so insulting that I doubt most people would admit to pursuing it. But in my experience, it is what people pursue -- and our education system trains people for it.

I became very aware of this when I started teaching computer classes. I was teaching applications such as Photoshop, Illustrator and Flash. Most of my students were upper-middle-class, educated, "smart" people. The majority were middle-aged.

Over and over, I heard people say, "I can't do this stuff. I'm just not a computer person." Now to some extent, this is true. These people were born before the Internet and the PC revolution, and their fear of the technology WAS a stumbling block. But the bigger stumbling block seemed to be that these folks couldn't handle basic problem solving.

The apps I taught mostly didn't hold your hand. For instance, if you want to make a photo look a certain way in Photoshop, there generally isn't a button to press. You have to think through the various tools and figure out how to combine them to create the look you want. That said, it's far from rocket science. I found that the moment I stopped giving people a formula that they could learn by rote, their brains turned off. It soon became clear to me that the problem wasn't new technology; the problem was that I was expecting people to use their brains in a way that no one else expected of them.

I started thinking about what these people did all day at their jobs. Gently, I asked some of them about what they did in their jobs. Many of them hand distinguished careers. How could they perform well at work without problem-solving skills? Answer: they don't need problem-solving skills.

It's not always obvious that these people don't solve problems (or puzzles), because many of these people are experts -- meaning that their brains are crammed with obscure facts. Our schools do very well at training people to learn facts*. At least when I went to school, memorization was pushed as a major intellectual virtue. We memorized the multiplication tables; we memorized the periodic tables; we memorized speeches form Shakespeare... Cultural literacy was pushed, too, though not as hard as memorization. No one was expected to really get into Shakespeare, but you were expected to know who he was and to have read one or two of his plays.

(*true, in America shocking number of people can't tell you the name of their congressman or the capital of North Carolina. But these people DO know the facts needed to get their specific jobs done.)

Pop-culture values reinforce fact-based intellectualism. A couple of years ago, if you'd asked people who was the smartest man in America, many would have said "the guy who won all that money on 'Jeopardy.'" (When I was a kid, there were many game shows on that actually required some problem-solving skills. These are almost non-existent. The shows are all about trivia now.) A "smart person" on a drama or sitcom is usually a guy who knows a huge number of facts.

I grew up around (humanities) academics, supposedly the ultimate smart-set. In my experience, they were coasting on memorized facts just as much as people in the corporate world. A professor would read every major German novel written in the 19th Century and all the critical writing about 19th-century German literature. Then he would spend his career passing on facts to his students. His "intellectual" work mostly involved keeping up with academic journals (learning new facts).

(From what I can tell, most G.P. doctors and most lawyers don't have to do much problem solving either. I do know that my doctor seems to be able to make a good living by doing the same formulaic tests over and over.)

Let me be clear that I'm not anti fact or memorization. Facts and rote learning are important. Facts are the building blocks you need. The are the tools you use when you problem solve. Problem solving is the next step. But it's a next step that most people don't take and don't need to take.

I don't think it's laziness. One can get by in our culture without problem-solving, so why bother with it? By get by, I mean that one can make a good living, have a big house, kids, etc. without having to solve intellectual problems.

And -- most important -- one can be a "smart person" (as our culture defines it) without solving problems. Most people want to be smart. They want to be seen as smart by others. Our culture sends a really strong message to them, which is "memorize a lot of facts and you'll be smart." My guess is most people think they ARE doing rigorous problem solving when they see something that needs to be done and have to search through their mental database to find the right fact or the right formula. I guess this IS a kind of problem solving, but it's the easiest kind. It's similar to solving a problem by searching on google until you find the answer.

When I was a kid, there was almost no problem solving in school. How often did the teacher just present us with a puzzle and say, "Here are some tools. Solve the puzzle!"? Almost never. One would think that MOST of education should be about solving puzzles, but in my experience, almost none of it is.

The exceptions (to a point) were Math and Science. But unless you're going into specific fields, you can quit taking Math and Science pretty early on in life. The other courses are easier and it's pretty clear you won't need Math and Science to get by in life. So why waste your time on it?

Meanwhile, the few people who stay in problem-solving fields move further and further from the intellectual norm: I program computers for a living. Which means I solve puzzles eight hours a day. I constantly have to create something from nothing, and I constantly have to learn new skills. Sometimes, I am so mentally exhausted that I can't do my job.

It was when I started discussing this with friends that I realized how different my career was from most of theirs. Sure, they often are exhausted at work. But they CAN get their work done. They say things like, "I was SO sick of filing today" or "Uh. If I have to grade ONE more paper!" But they don't say, "My brain just shut down and I was unable to figure out..."

I know this sounds snobbish. But I am not trying to diss other people or their jobs. My doctor may not do much problem solving, but I am grateful for his help. I am just saying that most jobs involve little or no problem solving. Mathematicians are from Mars.

I have been talking mostly about corporate and academic jobs. In reality, I think there's a lot of problem solving going on in America. It's just outside of the intellectual world. And it follows a long tradition. In America, our main problem solvers are farmers, football players, carpenters, etc. People who build things and who play games MUST solve problems or they fail. It's really weird, because most such people can't talk the intellectual talk. They don't know Shakespeare from Euler. So we don't consider them smart, and they aren't smart in the limited way we tend to define the world.

Meanwhile, the "intellectuals" are barely using their intellects.
posted by grumblebee at 11:13 AM on October 17, 2009 [460 favorites]


Math (and the basic hard sciences in general) is not something that can be debated, not something that has "two sides" that need explaining, not something that you can make up a bunch of woo-woo about to handwave away when you're not willing to think with precision.
I can't imagine equating "coming to a predetermined correct answer" with "thinking with precision." It's true that some people can't deal with ambiguity and can't think creatively, and those people tend to like lower-level math and science, because there are right answers. At the top levels, math and science are all about ambiguity and creative thinking, and there aren't any clear answers, but most of us aren't capable of doing math or science at that level. I don't think you're doing math and science any favors, however, when you depict it as a refuge for people who can't deal with the complexity and open-endedness of human existence.
posted by craichead at 11:18 AM on October 17, 2009


"2. It's binary. Either you're right or you're wrong. I'm sure this isn't the case when you get to the more advanced, theoretical levels of either math or science, but it takes many years of that binary experience before you're ready to debate theory. So neither subject felt creative in the least."

That's a little like saying, "there's no ambiguity about whether Hamlet said 'to eat or not to eat' or 'to be or not to be.' The latter is right and the former is wrong!"

The creativity in Math comes from the way you USE it. And you don't need to know a lot of Math to start using it creatively. The fact that kids don't is not Math's fault -- it's the fault of the education system.

If you've ever see kids programming in LOGO, you'll realize that they can use Math in all sorts of creative ways.

Another thing that people don't get is that Math is a deeply SOCIAL activity. When is something in Math or Science deemed true? When a mathematician or scientist convinces his community that it's true. He can't use politics or persuasion to do this -- he has to use proofs -- but in the end, it all comes down to convincing people.

We could argue the philosophy of this, but I'd say a very real way of explaining why Evolution is true is that Darwin convinced the scientific community to accept it.

Creating proofs is a deeply creative act. Often, there are multiple ways to prove the same thing. Once you know some very basic arithmetic, you can already start creating proofs. But math courses rarely (if ever) have students do this.
posted by grumblebee at 11:21 AM on October 17, 2009 [4 favorites]


This is really not limited to math and science. My dad's a philosophy professor; he gets the same responses. I'm a linguist; I get them too. I know other people who teach English and history and so on and I get the impression they all get them.

My sense is that it's not math-phobia in particular. It's a combination of anti-intellectualism, polite modesty and a sort of misplaced anti-elitism.

"Acting smart" is taboo in mainstream American culture. We see it as a kind of pride, and as a threat, and we don't tolerate it. Even at elite universities, or in accelerated high school classes, you'll see kids going out of their way not to be marked as a brain, not to be too good at what they do or too enthusiastic about the material, and a lot of us retain some of that attitude as adults. (Even very geeky adults tend to sort of flinch at the idea of showing off their smarts in less geeky company.) Claiming any kind of specialized knowledge can also be seen as rude, I suppose because we tend to feel like it's unfair to let any one person be an authority on something.

And so a lot of people develop a reflexive deference around intellectual subjects. "Oh, I wouldn't understand any of that," they say when anything intellectual comes up, the same way they'd say "Oh, I'm fine" if they tripped over someone's foot and "Sorry" if they tripped someone else — as a polite gesture and a way of making sure nobody feels threatened. Often it's a social lie. People say they're fine even if they're a little bruised; they'll say "sorry" even if they think it was the other guy's fault; and they'll profess total ignorance even if they secretly think the subject's kind of interesting, just to make sure nobody thinks they're showing off.

If you want to play along, the right response is to say "Hey, it's less complicated than you think" — another social lie, but we tell lots of those — and mention some tedious or unglamorous part of the job. I don't know what the drudge work in cosmology is, but there must be some. That establishes you as a regular modest guy, and not one of those snooty judgmental intellectual strawmen that everyone's scared of meeting or looking like. It's a ridiculous and irritating bit of song and dance that we wouldn't have to do if we didn't live in a culture that was so hung up about thinking; but we do live here, so there you have it.

I then began to think about people's ideas of a well-rounded, well-educated individual: he/she is expected to have read Shakespeare, but he/she is not expected to know the two postulates of relativity, or all three of Newton's laws, or what a derivative is.

Ironically, your alma mater takes a bit of the blame for this one. This is the Great Books approach to education in action — the idea that reading history and literature makes you a better person, while math and science ability are mere technical tools. But try telling someone at a party that you spent your Friday night reading Shakespeare, and you'll get the same response you get when you bring up cosmology. There's definitely still a tendency within some corners of academia to emphasize reading and writing over math and science, but outside academia it's all seen as weird and elitist.
posted by nebulawindphone at 11:43 AM on October 17, 2009 [12 favorites]


(Oh hi grumblebee. Yeah, what he said.)
posted by nebulawindphone at 11:47 AM on October 17, 2009


#1, in elementary school, we had to do those little sheets that were like a timed test of one minute to do all the problems on them. I was never, ever, able to finish them. Even the really simple ones! And I remember just being constantly berated about it to the point where I stopped going to classes and just hid under my desk. I wound up with a permanent pass to the guidance office and so I would just leave the room during that sort of stuff because the teacher couldn't deal with me just hiding and crying.

I have two early memories about Math and school.

1) It was 3rd grade and we were learning about sets. I thought I knew what a set was -- I was SURE I knew. I thought a set was a group of similar objects. (That's generally how we use the word set in English -- e.g. a set of salt and pepper shakers; a complete set of Beatles albums.)

The teacher asked us to give her examples of sets. One kid said, "All the students in this class could be a set." The teacher said, "Right." Another student said, "There could be a set of all the teachers, too." The teacher said, "Right" again.

Then this girl said, "The pencil sharpener and the flag pole could be a set." Oh Jesus, I thought. What an idiot! But then the teacher said, "Right!" And I felt like I had slipped into the Twilight Zone. If ANY objects could be in a set -- even totally unrelated ones -- how could "set" be a useful concept? At that point, my brain shut down.

The teacher never explained sets or how they were useful or beautiful. She just had us read a page of our Math book and then quizzed us. I wonder if I was the only kid who was confused.

2) The first time Math ever excited me was when I was in 6th grade. I learned Zeno's Paradox and was blown away by it. I had no one to talk to about it, so I went to my Math teacher, Mrs. Wilsey.

I made the mistake of accosting her in-between classes, when she probably was dying to go into the Teachers' Lounge and smoke. In a nerdy, quivering, excited voice, I said, "Mrs. Wilsey! I just read this paradox: if you want to walk across a room, you have to walk half way across it first. But before you can do that, you have to walk a fourth of the way, and before you can do that, you have to walk an eight of the way... if this goes on forever, you can never cross the room. And yet you can!"

Mrs. Wilsey stared at me for a minute and then said, "Get out of here!"

Which is when Math died for me. I honestly think my life might have gone in a different direction if she'd discussed it with me.
posted by grumblebee at 11:50 AM on October 17, 2009 [5 favorites]


My experience very much mirror's nadise's. I was a very bright kid in very advanced classes. I did fine in math until I hit pre-Algebra in seventh grade. Even though I eventually pulled a high C, low B sort of grade in the class by absolutely working my ass off, I didn't actually learn anything or understand anything. I did not have the knowledge base necessary to succeed in the future.

After that point, everything went downhill. I have never worked as hard at anything in my life as I did in my math classes the next few years. I had a couple of decent teachers and at least one terrible one, but it was basically like studying in a language I didn't know. I was slightly less terrible at Geometry than Algebra, but was essentially lost. My teachers either didn't want to bother or didn't know how to explain to me what I was doing wrong. They'd tell me to practice, but it doesn't do much good to practice the wrong thing over and over again.

There was never anything beautiful, intuitive, or relevant to me about math. I never once had that invigorating feeling where a concept clicks on in your brain and it's like a door opening with a whole new universe on the other side. It was just wrongness and failure. The only thing I ever learned from math was that there are some things that even when you work your hardest at them, you may not succeed and if you don't, your effort counts for nothing.

By Algebra II, I was frequently in tears in class from anxiety and frustration and the feeling of hitting the same damn wall of my own ignorance over and over again. One day when I went to my teacher's desk to ask, yet again, for help she sighted with frustration and said "Just go sit down Martha." I failed the second semester of that class and almost didn't graduate from high school because of it.

So, yeah, I hate math. Loathe it. I liked and did well at the less math-heavy sciences (Biology and Anatomy; I was a hell of a dissector) and loathed Chemistry (which was just a math class in disguise). However, I'm pretty impressed with people who work in advanced math-based fields; they excel at something I can't begin to understand and have work and life experiences that are very different from mine, so generally that's interesting.

The only time I have a hard time with people in math or science fields is when they don't take my work seriously. I'm obviously one of those humanities people. I'm a writer and editor who reads history for fun. I think words are beautiful and exciting; wrestling them into submission and putting my thoughts and feelings on paper and from there into someone else's head delights me.

I work with a lot of computer engineer types who often don't take what I do seriously at all because it seems easy and trivial to them. However, I suspect some of what you and I see as people looking down on our life passions is real, and some of it is our own insecurity.
posted by mostlymartha at 11:52 AM on October 17, 2009 [2 favorites]


But try telling someone at a party that you spent your Friday night reading Shakespeare, and you'll get the same response you get when you bring up cosmology.

Funny. This reminds of when my wife and went to Jamaica on vacation. About ten years ago, I recovered from my bad math education and became interested in learning the subject. But I never had time. Now, finally, we were going to lie on the beach and I had nothing I had to do. So I packed "Introduction to Calculus" along with the latest P.D. James novel and my iPod.

A couple of days later I was sitting in a beach chair, listening to the waves hitting the shore and happily learning about functions, derivatives and integrals.

Suddenly, I heard an angry voice by my side. I turned and realized the guy next to me was talking to me. He seemed pissed off. He said, "Dude, why don't you relax?"

I said, "Excuse me?"

He said, "You're on vacation. Why don't you put the fucking work away and enjoy yourself."

I said, "This isn't work. I'm reading this for fun."

He looked even more pissed, said "Whatever," and turned away from me. It was clear that I was offending him by reading a math book on the beach.
posted by grumblebee at 11:58 AM on October 17, 2009 [27 favorites]


Regarding the responses you get when telling people your profession: I don't think most of them actually hate math and science, I think this is just some THING people DO. They have to form some kind of response to your statement but they know nothing about astrology/cosmology.

This. People can be pretty fumble-tongued when they ask someone what they do for a living and they get an answer they're not expecting. Banker? Every layman knows what a banker does. Businessman? There's a layman's conception of what a businessman does. Doctor? The layman gets what a doctor does all day. A writer? You have an idea of what a writer does. But...a cosmologist? ....The layman doesn't quite get what you do.

So when the layman meets a banker or a businessman, they can make small talk about the economy. When they meet a doctor, they have an idea what a doctor does all day, so they can riff on that. With a writer, they can ask "oh, what do you write?" But....when you have only a sketchy idea of what a cosmologist does, then you flounder a bit. But that's not a function of "I don't get cosmology," it's a function of "I don't understand what that job entails." I mean, I got that a lot too when I was a stage manager, and that's not a scientific field at all. I would sometimes explain that I work backstage in theater, and they'd go, "Oh, so you move the scenery and stuff?" And I'd say, "actually, no..." and they'd just frown in confusion. They don't know what a stage manager DOES, they couldn't RELATE to it.

So I think it's more of a human reaction to "understanding people's work days in broad strokes so you can make small talk." It's easy to fake it with small talk when you have an idea what it is they do, but not so easy when you're meeting someone with a less office-oriented job.
posted by EmpressCallipygos at 1:28 PM on October 17, 2009 [3 favorites]


I'll echo a lot of the points other people are making in this thread:

1.) In the vast majority of my math classes... none of the teachers were ever able to explain to me WHY the functions and formulas worked the way they did. In many other fields, you can dissect an approach (such as how all the parts of an engine work or why farming wheat a certain way is better) and easily explain why society arrived at these approaches as the ones we currently view as "best". I never found a math teacher who was able to do this. If you asked why a formula worked the way it did, the answer was usually "Because thats the way it is." (which is like your parents telling you to clean your room "because I said so".. it doesn't help in any way, shape or form in your understanding of WHY). Also, it seemed drilled into my head that there was precisely (and only) one way to correctly solve an equation. That seems very much at odds with many other areas of life where there are endless variable ways to solve a problem. At any given time when I sketched out a equation incorrectly, the only thing my teachers could do was tell me it was wrong, but never WHY it was wrong.

2.) All through school my math and science teachers swore up and down that what they were teaching me would be applicable in my everyday adult life. (I'd try to ask them for specific examples - but none were ever offered.).... as it turns out, I pretty much don't use any math or science. I'm not necessarily pointing this out as a way of calling my teachers liars... but more so because I feel disappointed that no one was ever able to teach me math in a way that would be approachable and relate able. I barely passed algebra with D's.... because it was severely intimidating and foreign/esoteric... (those long strings of equations mean absolutely nothing to me. "Solving for Y" doesn't make my laundry easier or my fuel efficiency better).. but if the tests had been rewritten (maybe in the form of word problems) showing how the equations connected to real world concrete things.. then I might have had a chance to making a mental connection.

3.) I have to agree with what many others have commented here on the topic of rote memorization. Through much of my youth schooling - it was very obvious (even at that age) that the majority of teachers cared very little about igniting passion, teaching problem solving or critical thinking skills. Granted, I don't place all the blame on teachers - I also think our educational system has been constructed and refined to emphasize commodity learning and ease of testing/results. (and as many others have mentioned, critical thinking skills are unnecessary to achieve the average suburban lifestyle (graduate, buy a house, picket fence, SUV, etc -- be a cubicle worker who doesnt rock the boat). I'd go so far as to say that math and science have the potential to dramatically change our understanding of reality - and most people don't want that. Humans like predictability.

I'm currently trying to learn programming.. and I see a lot of parallels between math and programming. It's very structured, logical, esoteric and poorly explained. (the field expects ALL learners to learn in the same way without exception). I can see a lot of pedagogic mistakes being made (and passed down to each generation of learners) which complicates the field and leads to a lot of arguing and proselytizing. It's no wonder subjects like these are approached with apprehension and a pre-existing belief that you'll fail before you even try.
posted by jmnugent at 1:32 PM on October 17, 2009


Presumably the texts books have a hand in this as well. Takes an extraordinary teacher of any subject to overcome bad textbooks.

But that raises the question, where do they teach math properly? And how exactly do they teach it?
posted by IndigoJones at 1:47 PM on October 17, 2009


I think with many people it may be anti-intellectualism, but honestly, some of it may just be the passion and coherence with which you speak about your field. This might not influence one-liners or reflex responses, but when it comes to sustained conversation about what you do, a really engaging quantum physicist could make his work seem much more accessible and relevant than, say, a film-maker who fumbles explaining what his current project is about. You'll never convince someone who hates learning to like cosmology, but someone like Carl Sagan could convince thousands of laypeople to spend their time soaking up information about the universe.

Not saying that this is where you're going wrong, but it's something I've noticed in life.

(re: anti-intellectualism: I'll never forget when I picked up this Brain Quest Game and took a look at the tagline. It felt like someone had just punched my soul.)
posted by shaun uh at 2:14 PM on October 17, 2009 [1 favorite]


Best answer: But that raises the question, where do they teach math properly? And how exactly do they teach it?

The problem with this question -- which is the one everyone asks about education -- is that there's no "way" to teach it (or anything) properly. Good teaching is never formulaic. You never hit on the right way to do it. There is no right way.

ANY teacher who repeats what he did last years is a BAD teacher. It may not be his fault that he's bad. It may be "the system." Nonetheless, he is bad.

A good teacher has a goal in mind and realizes that, in order to meet that goal, he must see each student as an individual. He can't teach a whole class. He can only teach each individual student in that class. Each student has his own strengths and weaknesses, his own baggage, and his own style of learning. So there's no general technique a teacher can use. From my years in the classroom, I would say there's not even a general technique that works with most people. We need to stop pretending that there is.

Good teachers are ace problem solvers. They see each student as a puzzle and spend the school year trying to crack all of the puzzles. They ask questions, make observations, try different approaches, etc.

It is SO tempting to answer this question by saying, "People learn visually, so math should be taught with pictures." Or "People learn by doing, so math should be taught via experiments and puzzles..." But as long as we're trying to figure out how math SHOULD be taught, we're on the wrong track.

There are many stumbling blocks to effective teaching:

- Most people think too small. To them, a debate about education is about which text book to use or whether more time should be spent on Algebra or Geometry. See above for why this is missing the forest for the trees.

- It's really easy to become a teacher. Unfortunately, good teaching requires our top creative minds, not "just anybody."

- Teachers -- like most people -- tend to coast. They find some approach that works for some of the students, stick to it, and decide that the rest of the students are dumb. As long as they don't pass kids or flunk too many kids, they are allowed to get away with this. It's expected. Teachers are not expected to sit up until 2am every night, trying to figure out how to best reach a particular kid.

- Teachers are underpaid. I almost didn't list this, because it's what everyone always brings up, and I don't agree that it's the major problem. I'm not convinced that if we started paying teachers 200K a year we'd have much better teachers. I think it would help somewhat, but not nearly as much as people think it would.

There are many professions that don't pay well. Many artists and scientists make a fraction of what they should make. Yet they hurl themselves passionately at their crafts. I think the problem is less about pay and more about how little our culture values teachers. It's about our low expectations for them. Yes, if we had higher expectations, we'd pay more. But saying that teachers should get paid more is placing the cart before the horse.

- The basic structure of our educational system is at odds with learning. Grades, required text and exams all hurt more than they help.

- Children are born scientists. They are naturally inquisitive and love learning just for the sake of learning. Somehow, this passion fades for most of them. Why? We urgently need research in this area? Is this just a natural part of growing up or does our culture arbitrarily encourage it? I suspect the latter, but I don't have evidence for it.

- Truth is, many intellectual subjects aren't needed in a nuts and bolts way. You can live a happy and productive life without ever reading or seeing a Shakespeare play, without doing any Calculus, without listening to Mozart, without understanding Cosmology, etc. We need a deep and healthy debate about why we teach these subjects and whether or not they should be required. Whereas mostly what we hear now are platitudes, bullshit and "because I said so."

I am very pessimistic that our educational system will stop being broken. Most debates are about how to distribute resources. How much should teachers be paid? Should we teach this or that? Should we use this textbook or that textbook?

What's needed are not more resources. (Well, they are needed, but they are not the main things needed). If that was the problem, I would be more optimistic. Given the right political climate, that's at least theoretically solvable.

Alas, what's really needed is a paradigm shift in the way we think about education. That's a toughie.
posted by grumblebee at 2:21 PM on October 17, 2009 [20 favorites]


Having said all that, chicago2penn (the OP), why don't you do something about your problem? BECOME a teacher. I don't mean you should quit your research and go into the schools. I mean you should learn to explain what you do. There are wonderful people who do this for us: the Carl Sagans and Richard Dawkins of the world. Folks like this are too few and far between.

You don't have to write a book. You just need to give a big think to how to communicate what you do and what's exciting about it. And then next time you get a blank stare, say, "I know, it's not the sort of profession you hear about every day, but let me explain what I do..."

Some people will get turned off. (I already didn't care, and now he's going on and on about it.) But many others will be grateful to you for explaining something complex in a simple, yet non-condescending way.

If scientist don't help us understand Science, who will?
posted by grumblebee at 2:25 PM on October 17, 2009 [3 favorites]


I have a BS in math, but I went to a hippie school for grades k-4, so when I entered public school in the 5th grade I could only do math on a 1st grade level.

By 7th grade I was in the advanced math classes in my middle school. I think part of why I love math is because I got to skip all the rote memorization and punishing arithmetic races that people have described upthread. I got to go pretty much straight into algebra and never really looked back.

My little brother entered public school at the same time I did, in grade 2. In the hippie school he loved math and seemed to have a lot more promise than I did, but by grade 5 he was struggling to pass his math classes at all.

Anecdotes, I know, but I have considered this question pretty thoroughly myself, as my degree tends to get nearly the same reaction yours does, and it always embarrasses me.

For what it's worth I am no more than indifferent at arithmetic. I hate it when people interpret "math degree" to mean "savant-like mental calculator."
posted by ZeroDivides at 2:35 PM on October 17, 2009


What turned me off of math, even though I always did well in math classes, was the structure of the sequence of math courses in middle and high school. You have a year of Algebra I, a year of Geometry, a year of Algebra II, a year of pre-calc/trigonometry, and then finally, Calculus--which is seen as the culmination of all your learning. I know calculus is important and has many "real-world" uses, but they tend to be more esoteric and abstract than the real-world uses of geometry or statistics. And even when I was in high school, I was pretty sure I'd never follow a career path that required me to use calculus... so it felt like much of my math education was for naught. If American high-school math focused more on disciplines like geometry and statistics, whose usefulness to solve everyday problems is more self-evident than that of calculus, I feel like fewer students would hate math.
posted by clair-de-lune at 2:35 PM on October 17, 2009


there's no "way" to teach it (or anything) properly

I should clarify - the big beef seems always to be, we do science ed so badly in America.

Okay, fine. My question becomes, where, where in the world do they do it well? Germany? Asia? From all I hear, they tend to rote, which is not well regarded, and yet, said Asians are a cliche of math/science superiority, possibly deserved for all I know.

Another reason I asked about the text books.
posted by IndigoJones at 2:48 PM on October 17, 2009


From all I hear, they tend to rote, which is not well regarded, and yet, said Asians are a cliche of math/science superiority, possibly deserved for all I know.
I don't remember the details, but I read somewhere that it's a big myth that Asian countries teach math by rote and that places like Singapore focus much more on math concepts and much less on rote memorization than the U.S. does.
posted by craichead at 3:00 PM on October 17, 2009


clair-de-lune just hit on one of the problems people have with math: they're generally taught the math before their taught what it's good for. That's fine for math majors in college who just love the subject, but it's hard to motivate most people if they don't know any real-world uses.

Calculus? You can't make it halfway through the Laws of Motion before you hit a derivative; you can't throw a ball in the air without watching "d2y/dt2 = g" in action. Practically any law of physics ends up having time derivatives or spatial derivatives or both in it, or is expressed as a conservation equation with volume and surface integrals.

But that's all physics class stuff, and sometimes it's vector calculus stuff that's hard to simplify down to a one-variable first-semester case, so you won't hear much of it in intro calculus classes, not unless your school is particularly interdisciplinary or your textbook is desperate for word problems. It's as if we expected kids to learn that 2+2=4 without even explaining a connection to "2 apples and 2 apples make 4 apples".

I'm not sure what to do about that, either. We can rely on kids already knowing what apples are, without waiting until they're in "fruit class" to get to it. The same probably isn't true for electric fields and continuum stress/strain; to learn what the math is useful for you have to already know the math.
posted by roystgnr at 3:19 PM on October 17, 2009 [2 favorites]


I don't think people generally hate science per se, but they're pretty dismissive of math. And as previous commenters have said, it's really because of how the subject is taught. In the United States, at least, it is taught entirely by rote. It is taught in a vacuum, by people who do not really understand it, and who therefore manage to convey neither its sublimity nor its importance as a vocabulary and a toolset for the exploration of nature. I did not continue much past calculus, and at the time I had no sense of what I was abandoning. I only knew that doing math felt a lot like complicated cooking, only without the production of anything delicious at the end.

It was only much later, when I went back to school and took some physics, some physical chemistry and some other math-heavy science, that I felt how bounded I was by my own lack of fluency. Because I had trouble with the math, I would sometimes try to understand some of the more esoteric concepts descriptively, but without solving the equations, without being able to grok things symbolically, I often felt I wasn't really getting it at all.
posted by killdevil at 3:24 PM on October 17, 2009 [2 favorites]


I was a student in an elementary teacher education program back in the eighties. After several of my classmates mentioned that they sucked at math and/or hated math I did an informal survey and asked about 40 future teachers how they felt about math, only one said she loved it.

It really made me angry that we were going to unleash all of these mathphobes onto innocent young children and many of those children would become mathphobes themselves.
posted by mareli at 3:59 PM on October 17, 2009 [1 favorite]


And how exactly do they teach it?

The best math teacher I had before coming to college was my 8th grade Geometry teacher. The more deeply I get into the subject at the collegiate level, the more I appreciate how really good a teacher he was. A lot of people become math teachers because they think "I want to teach. What should I teach? Math is easy for me, I guess I'll teach that." Mr. B was the other way around -- he knew his mathematics, and decided he'd be more fulfilled as a teacher than an accountant. I'm sure that, more than anything, is the key to his being such a great teacher.

I'm not sure how deeply educated he was in research-type math, be he knew it existed and loved it. To the best of my recollection, he was the only teacher I had who made any kind of reference to the idea of mathematical research; he taught us a bit about the fourth dimension, for instance, and I'm pretty sure he was the first person to explain Fermat's Last Theorem to me.

But the thing I will remember him most for is something I didn't really grok at the time. Sometime during the second semester, he gave us a project. We got a map of a town with traffic patterns marked on it, including spots where there tended to be jams. We were supposed to modify the traffic patterns to eliminate the jams.

I wasn't too sure that it had much to do with geometry, but it was a fun problem and everybody seemed to have a good time with it. Just a few weeks ago, I was reading something for my Linear Programming class about traffic patterns and it suddenly dawned on me: holy shit, this is exactly like Mr. B's traffic project. He gave us that assignment because it is exactly what a huge number of actual mathematicians do all day; he just scaled it down and let us run loose on it.

That was what he was so good at, showing how much fun "real math" can be, even if he didn't outright tell us that's what he was doing. Most of all, he understood that there's a whole world of mathematics out there, that it's something more than just "equations and stuff", and he made sure we knew that, too.

From weapons-grade pandemonium: math contains no love or emotion, as do art or music or literature.

It pains me to read this; you make it sound so lifeless. The beauty doesn't come without passion. The answers aren't pulled ready-made out of a book. They have to be created. In most high school mathematics, you don't see that process of creation, but it is still there -- everything you ever learned in a math (or science) class was ultimately the product of someone's mind. All of it had to be thought up by somebody, often a somebody who poured months or years of their life into it, banging their head against dead-end after dead-end, until they hit upon the right path at last.

What so few people understand is that mathematics isn't just lying around to be picked up off the ground, that you can't get a theorem by following a formula. 10th grade algebra class might be all about following rules, but real mathematics is all about creation. Creative work absorbs people like nothing else, and that is where the emotion in it comes from; the creator is so tightly involved in his creation that little pieces of himself rub off and bury themselves inside the work. A third party (a listener, a reader, a viewer) can then find these pieces, and be moved by them. This happens just as much in math as it does in music; most people just aren't taught to look for it.
posted by Commander Rachek at 4:12 PM on October 17, 2009 [8 favorites]


Kylej pretty much says it: you need arith before doing the more complicated; the girl/boy balance has probably "improved" if not swung the other way in some places; unlike history or literature, you need to understand level X before level x+1. And, as many have said, math is learned but there is also a heavy "ability" component.

I would add: there is a cultural distaste that reflects the old american anti-intellectualism. That distaste is reflected in the mathphobic (with many exceptions) elementary school teachers, who generally are the least academically accomplished (with many exceptions) of professionals as measured by standard tests. It extends to jr high and high school where many teachers (with a broad class of exceptions) are draftees to the subject lacking a foundation in the subject even of a chem, physics, or engineering major, not to speak of a math major or advanced degree.

To teach effectively, in a way that conveys the essence and builds intuition, not just the facts, you really need to know the subject at least one level deeper than what you are teaching. That allows the teacher to explain why a function or method works the way it does, not "just because." That allows the teacher to creatively interact with the class and individual student.

The problem is fundamentally circular with marginal teachers creating discouraged students, from which come the next generation of marginal teachers. It is interesting that other parts of the world do not have this problem, suggesting a primary cultural malfunction.
posted by Kevin S at 4:45 PM on October 17, 2009


We can blame teachers, we can blame textbooks, we can blame the culture and its anti-intellectualism, but I don't think there are solutions lurking in these diagnoses. A lot of the creative math techniques were tried and are now considered failures. I know, like true communism and true capitalism, we can argue that they really haven't been fully tried yet but that's a cop out.

Katherineg is closer to it when she says different people have different brains and like different stuff, and a lot of the humanities and social sciences are entertaining and immediately relevant.

The people who will appreciate what math has to offer are a small subset and it could be increased by "better" teaching, but I doubt that it can be increased significantly, or else only at the cost of losing some other segment of the population. And, yes, ultimately teaching can't be one size fits all, but as scientists, we can only set up experiments with those kinds of hypotheses (see the discussions about CBT as a proven therapy method.)

G. H. Hardy wrote a book in the 40s called A Mathematician's Apology in which he tries to make a case for mathematics independent of its applications. It's not a case that has generally been accepted outside of the world of math geeks, and when I was younger, I never understood why.

But now it's obvious to me why and it's not because of the teaching or the culture so much. My first clue came when I tried to teach a girlfriend what it meant for one note to be higher than another. The concept of a linear order to pitch didn't apply in any meaningful way to what she heard. And even if I could teach her to apply it, it wasn't natural to her experience. And it never became so. Her brain was very different.

Another clue came from the flip side--how I couldn't "get" certain ways of seeing the world that seemed to be self-evident to others. Our ability to empathize is limited. Those on the outside of the bell curve will be diagnosed as thought-disordered, autistic, learning disabled, wierd. And those sufficiently different from how we are see us the same way and wonder how we can be cosmologists. It's not impossible to cross these empathic divides. It's just that few bother trying and it's a lot of work to do even a passable job.
posted by Obscure Reference at 6:44 PM on October 17, 2009 [2 favorites]


In my experience many people who hate math and science also think it is stupid, worthless, and emotionless. This is untrue; those who think it are ignorant. It is perfectly acceptable to not want to be a mathematician or scientist because you think other subjects are more interesting, but it is important to at least realize that math and science are just as important and relevant.
posted by lucy.jakobs at 9:20 PM on October 17, 2009


I was pretty much a moron at math from the first day they handed me the worksheets. (Note: I suspect I have dyscalculia, but never even heard such a thing existed until I was out of school.) I did the same thing with those time tables tests that was mentioned above. It took a looooooooong time for me to grasp basic math, and right around when I did, oh, hey, here comes high school. Oh, and it probably also doesn't help once the math gets so hard that your parents can't help you do it any more (a lack of math ability definitely happened with my mom and my aunt as well). And when before that, they start screaming at you in frustration because the only times table you can memorize is 6+6=36.

I took algebra one over two years' worth of time (my school's version of "math for morons") and just had no clue. I couldn't remember anything the teacher said from day to day. Yes, I had PEMDAS memorized and I still didn't know how I couldn't come out with the right answer. Oddly enough, I was actually pretty good at geometry/chemistry, but unfortunately I spent a LOT more years of school stuck doing algebra, spending every damn lunch and free period getting tutored by the teacher, and the teacher being fed up to hell with me never getting it. I have been told that there are two kinds of people in the world, those who are good at algebra, calculus, and pretty much every other kind of math, and those who are good at geometry....and frankly, the world was not designed for those people.

Oh, you know what I always thought was weird? The most advanced math class in my school was essentially "here, come in at 7:30 a.m. and read a math textbook and teach yourself." What the heck is that? I strongly suspect that for some people it is just intrinsic knowledge.

I made sure I went to a college that didn't require me to do math as long as I avoided doing a science major. I would have liked to have done science in college (beyond the token classes), but they all required calculus and there was no way in hell I was going to get it. I just had to accept that I was an idiot and was never going to do well in anything complicated in life, period. There are a lot of people out there like me, even if they didn't have dyscalculia. (Note: dyscalculia tends to occur more in females. Now you know why Barbie thinks math is tough.)

What is so scary about math is that a lot of us Just Don't GET IT, no matter how much frigging tutoring we got. We didn't have fun learning. We got browbeat with how stoopid we are from first through twelfth grades. It's a sore spot. If you want to talk about your work, it's just gibberish to us anyway, you know?
posted by jenfullmoon at 10:52 PM on October 17, 2009 [2 favorites]


Something I learned recently is that, much like reading, math & math concepts are understood better at varying developmental ages for different people. Some kids learn to read in first grade, some in second, and most kids have it figured out by third grade (wild generalization). I was a kid that genuinely didn't get math & my second grade teacher railed on me about it until I cried, called me stupid, told my parents I was intentionally being difficult, told them I was a daydreamer, etc, etc. My dad majored in math & worked with computers until he retired a few years ago and every night, from second grade until I stopped taking math at the end of algebra 2, he sat with me, patiently checked my homework, and tried his darnedest to help me get it. He never gave up (meaning in geometry which I *really* didn't get, I took tests two, three, sometimes four times to the eternal dismay of the teacher who finally gave me a "courtesy D" to get me out of the class -- and it was the hardest damn D I ever earned, and we celebrated it at home like it was an A). There were never any light bulb moments, it never just clicked as 'oh, well that makes sense!' It was always a sisyphusian exercise.

Now? I use math all the time, on the fly, estimating costs, creating useful metrics, using numbers to show what my team does and figure out where we can improve, and what systems are inefficient that require reworking. I love spreadsheets (OMG, did I just admit that? Well, it's true. I do, so there!). I don't think I really got it until my late 20s, early 30s. Sometimes I practice what I call "optimistic math" where I mix something up & get a really exciting result that I think is awesome and I tell everyone about it and then I find I mixed it up & have to go tell people that I messed up, and that's not so fun, but it's more the being wrong part I don't like than the processes I go through to come up with the numbers. That said, I still suck at math -- when I took the GREs again to get into my graduate program, I studied hard, and the general wisdom is that you can't improve your scores more than about 10% in each area. I studied hard and improved the reading/language part by about 10%, math by about 10%, but my logic score improved more than 60% (this after having had my BA for about a decade). So I wonder if there's a developmental component of math that requires a certain level of logic and reasoning skills that aren't taught concurrently to the math? Or math is thought to teach the logic and reasoning skills required, but... obviously it doesn't work that way for all of us.

And I have a daughter. Who has been blessed with kind teachers who are patient and do their best to explain math to her and... she just doesn't get it. And I get it (so far, but then, she's only in fifth grade). And I can't make it make sense to her anymore than anyone could make it make sense to me. I get it, and I remember so clearly hearing others say what makes perfect sense to me now at her age and it made no sense to me at all. It's as frustrating to me now to be on the other side because I remember so clearly being where she is and I have No Idea how to bridge that gap, except to know that her brain will develop and she'll get there. She's smart, she's sweet, she's funny, we'll probably do math as a family until she stops doing math (or her little sister stops doing math), and... some day it will mysteriously make sense for her too. But for me at that age, and for her, yes. Math had/has big scary teeth or something.
posted by susanbeeswax at 11:47 PM on October 17, 2009 [2 favorites]


Someone upthread said that there's a profound anti-intellectual trend in America. I agree, but I think it specifically takes the form of A DISDAIN FOR PROBLEM-SOLVING SKILLS IN ACADEMIC SUBJECTS.

I not only favorited grumblebee's comment, I'm going to print it out and keep it on my desk.

I've worked with computers my whole career - ranging from programming to web design to IT / networking support to teaching and writing books about all of the above. When I tell people what I do, I get exactly the same "Oh, wow, I could NEVER do that!" responses that you get about cosmology -- which is strange to me because I consider computers far more logical and solvable and internally consistent than the ideas you work with.

In fact, despite my wife's working through a math degree and later teaching math, I always got far more wide-eyed confusion about my job than she did.

I think "disdain" is a bit too strong in the comment above. I'd say it's more a matter of confusion. People just aren't taught problem-solving skills, and so they're an alien concept. Tell people you build houses for a living, and people instantly identify with you, even though they could never do your job. The idea of a house appearing where there was an empty lot before makes perfect sense. But tell them you solve problems, especially in abstract worlds like cosmology or computers, and they end up with nothing to talk about except their own stupidity.

I'm not saying they're actually stupid. I think there are far less actual stupid people than most of us imagine. I've heard the "ooh, computers are so hard, how do you DO that!" comment from people who maintain incredibly complex fantasy basketball systems, and people who do killer sudoku puzzles just because they're bored, and people who run multi-million-dollar companies. I'm sure they could do what I do if they were trained, but without that training (and the desire to stare at a screen for 12 hours a day) they're left in the dark.

Some people ARE trained with problem-solving skills. I used to do IT support for a company that employed lots of diesel mechanics, and in the smaller branches the people responsible for entering data into computers were current or former mechanics themselves. I found that the good mechanics could often work with me to find a solution to their computer issues despite knowing nothing about computers. Much better than the accountants who worked all day with computers. Much better than some of the programmers I worked with in a different job, for that matter. No surprise - troubleshooting complex systems is what mechanics do for a living too.

On top of the problem-solving issue, there's the idea of dealing with levels of abstraction - another skill most people aren't taught, and something math and most sciences and computers have in common. That's why the car mechanic can tell people what he does for a living without being treated like an alien...
posted by mmoncur at 3:05 AM on October 18, 2009 [4 favorites]


I was pretty good at maths during my early years of school and represented my school in maths competitions, etc. Unfortunately I found all the work that my peers were doing in maths class extremely boring, and I wasn't allowed to do different work. So, I pretty much checked out for the next several years, until I got to high school. And at that point, the maths was stuff I actually didn't know, but I was so out of touch with how to learn maths that I never really caught back up.

Additionally, most of the maths teachers I had only wanted us to memorise and apply formulae so that we could pass the exams, and didn't care to take any time to explain them. Since I'm someone who generally needs to understand why a procedure takes the form that it does in order to remember how to apply it, this didn't really work for me. I also would have liked more practical applications. Statistics was always much more interesting to me than calc in large part because it tended to be taught with more real-world problems.

Then, after not taking maths at all my last two years of high school (having decided for the above reasons that maths classes were bullshit), I ended up taking an intro maths class for uni (it was compulsory for the degree I initially chose). I was of course hopelessly unprepared and I still had never really learned how to learn maths, and I failed the class. This further put me off maths, and I am just now trying to learn it by myself, in order that I could understand certain scientific topics I want to learn more about. In retrospect I wish I had had the motivation as a child to do more interesting maths work outside of class, so that I could keep my mathematical learning momentum going, as it is all much more work to learn it now than it ever was back then.

Science, though, I love. Wasn't particularly into the classwork in HS science but I have always read and still read books on science topics that interest me in my own time.
posted by lwb at 4:37 AM on October 18, 2009


Because maths is "you get it or you don't", or "you love it it or you hate it" subject. In fact, this is reflected in exam results in my uni. I was searching through the results of Theoretical Physics modules 1 and 2 of the Part II Physics Tripos the other day, and found some interesting conclusions. The majority of results were split into 2 camps; the first in the 30-40% region for a 3rd class/fail, the second in the 70-80% for a 1st class. It is clear to see that if you "get" it, then maths is probably something you enjoy. On the other hand, if you don't, you will probably hate it. It is just the nature of the subject.

A poster above said it was binary; that the answer is either right or wrong. Actually, this is pretty much completely true up to undergraduate level at university. Given a problem, there is usually one route to the correct answer via logical deduction. Sometimes, there are multiple methods, but this is rare. Unfortunately, creativity is not required. To reach the level where creativity is required, I am afraid that you'll need to do a PhD in maths.

So the simple answer to the OP's question is that maths polarises people. If you simply do not "get" maths, you will not be good at it. People don't tend to like things they suck at.
posted by dragontail at 5:50 AM on October 18, 2009 [1 favorite]


Why do elementary school teachers teach multiplication tables for 6 years and call that math?

I teach elementary school and I have never seen any of my colleagues teach math this way.
posted by HotPatatta at 6:21 AM on October 18, 2009


I read somewhere that it's a big myth that Asian countries teach math by rote and that places like Singapore focus much more on math concepts and much less on rote memorization than the U.S. does.

Interesting. I will try to look into that. Many thanks.
posted by IndigoJones at 6:33 AM on October 18, 2009


You've already selected best answers, but while I was driving last night, I thought of the perfect way to explain how confusing math is to me, even though I wanted to understand it.

Imagine I give you a box of Scrabble letters. I tell you, "You can put these together into strings that create words. You can string those words together into units we call sentences. Then, a certain number of sentences creates a paragraph. Now, go write Moby Dick. Don't get any of the words wrong."
posted by headspace at 8:12 AM on October 18, 2009 [9 favorites]


I'm a bit late to the party, but there's a great book, by a university of Toronto math prof named John Mighton, called "The End of Ignorance." In it he talks about the success of a math program he created called JUMP math. He developed it over a 20 year period while he was voluntarily tutoring kids. He turned it into a non-profit that works with school boards around the world and he still tutors kids for free while training more tutors. He claims that anyone can learn math given the proper instruction. He also says something really interesting in the intro to the book that chicago2penn hints at. He say that no one would happily and casually admit to being illiterate, yet so many adults gleefully admit to anyone who's listening that they are mathematically illiterate. It's almost as if it's socially acceptable not to be able to perform basic math.
posted by trigger at 9:56 AM on October 18, 2009 [2 favorites]


I was taught Arithmetic badly, then was taught Math during the New Math years, and was not well prepared to do algebra. I'm from a family there books were most important, with a healthy side order of science. (Thanks, GrandDad.)

I find Math to be difficult and interesting, but I was so far behind that I never caught up. I still love science, and did my best to encourage my son to learn science.

In my High School, it was considered quite odd for girls to take Calculus. I'm happy to hear that progress has been made. Interesting thread; thanks for asking.
posted by Mom at 11:33 AM on October 18, 2009


Unfortunately, creativity is not required. To reach the level where creativity is required, I am afraid that you'll need to do a PhD in maths.

This is absolutely incorrect. I'll grant that creativity is not (always) required to get a BA or BS in math -- it is probably possible to skate through some or most math programs by regurgitating what the professor said. But you definitely don't need a PhD (or any other special certificate) to do creative mathematics. There are plenty of problems that require creative solutions that can be understood by someone with no more than a decent understanding of basic algebra or geometry. For instance, proving the Fundamental Theorem of Algebra takes some mad cleverness, but both the problem and proof can be explained to high-schooler. The semester I took Intro to Proofs, I and several others in my class became involved with research while we were still in the course. If that's not a low entry barrier, I don't know what is.
posted by Commander Rachek at 11:45 AM on October 18, 2009 [1 favorite]


There's a trend in this thread that is familiar but deeply disturbing to me. It involves people claiming that they just can't get Math -- that their brains are not built for Math -- followed by reports of being abused by teachers. To me, this is like saying, "I don't like sex. I'm just not wired for it. I learned that about myself when I was raped at the age of fourteen."

Many people here will feel like my rape analogy is over-the-top. It's true that one doesn't feel educational abuse in the immediate way that one feels the pain of assault. But I believe that bad teaching is every bit as abusive as physical or sexual assault. The effects of bad teaching tend to haunt people for their entire lives. Most people don't feel haunted; rather, they blame themselves (sound familiar?). The say, "I'm just stupid that way."

There are two main hypotheses here:

1. Some people just aren't built for Math.
2. Math is ruined for some people by bad teachers.

My guess is that the truth is complex and involves both factors, but the test tube is so dirty that it's impossible to tell.

The people who will appreciate what math has to offer are a small subset and it could be increased by "better" teaching, but I doubt that it can be increased significantly.


The key is that you can't truthfully know that someone is just not built for Math until you've proved that their teachers made every effort on their behalf. Only after someone has been a recipient of excellent teaching and STILL doesn't get Math, is it safe to say that his brain is not built to get Math.

susanbeeswax: I was a kid that genuinely didn't get math & my second grade teacher railed on me about it until I cried, called me stupid, told my parents I was intentionally being difficult, told them I was a daydreamer, etc, etc.

That is common but criminally bad teaching. How is a child supposed to learn with a teacher that rails on her and calls her stupid?

My dad ... sat with me, patiently checked my homework, and tried his darnedest to help me get it. He never gave up ... There were never any light bulb moments

And I have a daughter. Who has been blessed with kind teachers who are patient and do their best to explain math to her and... she just doesn't get it.

susanbeeswax's dad and her daughter's teachers sound like lovely, kind, patient people, but I suspect that they are bad teachers. Her dad was a bad teacher because he wasn't teaching -- he was trying to help her complete her crappy homework and prepare for horrible tests. (It's not his fault he was a bad teacher). Her daughter's teachers are bad because they are repeatedly trying to "explain" math to her. Clearly that doesn't work: "she just doesn't get it." Good teachers don't just keep trying to pound the same nail with the same hammer.

The subject is Math. It's NOT a specific textbook. If a textbook is failing to help a student learn a subject, the textbook should be discarded in favor of some other approach.

If someone doesn't understand an explanation, it's not good teaching to just explain it again. And then again. And then again until you're sick and tired of the "dumb" student who can't get it. It's not good teaching to just keep giving the confused student more and more exercises and telling them to practice.

And it's terrible, abusive teaching to EVER show impatience, anger or disdain for a student who can't get something. Not only are you failing to teach the subject now, you're also pretty much ensuring that the student will never want to attempt the subject again.

I am making assumptions (based on experience) about these teachers, but I'd like to ask susanbeeswax this:

Do her daughter's teachers try using pictures? Do they try using physical objects (lego blocks or whatever)? Do they try using music? Do they try using videos? Do they try ten different textbooks? Do they try games?

Before and during Math training, did her daughter's teachers teach problem-solving skills? Did they teach how to break down a problem into small steps; how to brainstorm; how to research; how to collaborate; when to take a break?

Unless a teacher has tried all these things -- and more -- with a student who "just doesn't get it," he has no right to say that the student "just doesn't get it." Rather, he should say, "I'm just not doing my job. I am failing that student."

Let me be really clear in stating that many teachers can't be this creative -- even if they want to and have the skills. They are hemmed in by all sorts of constraints: financial and ones imposed by school systems and governments. But even if bad teaching is not the teacher's fault, it's still bad teaching. And it's certainly not the student's fault.

When I was a teacher I failed many, many students. That's the nature of the job. When a student wasn't making progress, my assumption was ALWAYS that I was failing. This was probably untrue sometimes. There were times when I was probably doing everything humanly possible. But it's dangerous to EVER decide that it's the student's limitations that are causing the problem. Once you decide that, you're off the hook as a teacher. You can stop trying. And the moment you stop trying, you risk missing out on a way you could help the student.

One more thing from susanbeeswax:

Something I learned recently is that, much like reading, math & math concepts are understood better at varying developmental ages for different people. Some kids learn to read in first grade, some in second, and most kids have it figured out by third grade

I agree. This is a proven fact of human nature. Since schools have to deal with humans, any school that doesn't take this into account is a bad school. Again, the school may not have the resources to allow each student to learn at his own rate, but that's not the student's fault. Before saying "he just can't get math," we need to be 100% sure that we don't mean, "he just can't get math at this age, but he might be able to get it in a year." That's generally not how it works. So people get labeled (and label themselves) bad at math in whatever year the subject is taught. They carry that label until the ends of their lives. This is a massive failure on the part of school systems.

If a teacher does what most teachers do, we can't say that the student "isn't capable of getting math." At most, we can say he's not capable getting math given the way it's being taught right now at this school. Is the student to blame for that or is the school to blame for that? Who is more stupid, the student or the school system?

A lot of the creative math techniques were tried and are now considered failures.

It does not work to "try a creative technique" with the whole class. Yet in my experience, this is how creative techniques get tried. Someone comes up with a new way to teach math, convinces a teacher or system to try it, and they do -- they try it with the whole class. And it fails. Of course it fails. All systems will fail, creative or not. THERE IS NO SYSTEM TO TEACHING. The creative technique that works is the one that is adapted uniquely for each student. Until we are blue in the face, we can say that such individual attention is just not possible due to bad funding or whatever. Too bad. It's the only thing that works.

If a fire department doesn't have the funding to buy trucks, ladders and hoses, a lot of houses will burn down.

odinsdream: The entire class could only go to lunch once everyone had filled out the sheet completely. I could not do this quickly, if at all. It was extremely difficult for me to do. This meant that my name was always at the end of the board, in the front of the class, without any time associated with it. Everyone was always 15 minutes late to lunch because of this. ... everyone else in the class complained, yelled answers, tried to be helpful, teased, etc.

TEACHING FAIL.

sprose: in elementary school, we had to do those little sheets that were like a timed test of one minute to do all the problems on them. I was never, ever, able to finish them. Even the really simple ones! And I remember just being constantly berated about it to the point where I stopped going to classes and just hid under my desk. I wound up with a permanent pass to the guidance office and so I would just leave the room during that sort of stuff because the teacher couldn't deal with me just hiding and crying.

TEACHING FAIL.

nadise: the only answer I ever got when I asked teachers and adults how to approach learning it was practice. Well, if you do the same thing over and over, you'll get the same result over and over. If that result is failure, that's not fun.

TEACHING FAIL

mostlymartha: My teachers either didn't want to bother or didn't know how to explain to me what I was doing wrong. They'd tell me to practice, but it doesn't do much good to practice the wrong thing over and over again... By Algebra II, I was frequently in tears in class from anxiety and frustration and the feeling of hitting the same damn wall of my own ignorance over and over again. One day when I went to my teacher's desk to ask, yet again, for help she sighted with frustration and said "Just go sit down Martha."

TEACHING FAIL

jmnugent: none of the teachers were ever able to explain to me WHY the functions and formulas worked the way they did. ... If you asked why a formula worked the way it did, the answer was usually "Because thats the way it is."

TEACHING FAIL

jenfullmoon: I spent a LOT more years of school stuck doing algebra, spending every damn lunch and free period getting tutored by the teacher, and the teacher being fed up to hell with me never getting it.

TEACHING FAIL

lwb: Additionally, most of the maths teachers I had only wanted us to memorise and apply formulae so that we could pass the exams, and didn't care to take any time to explain them. Since I'm someone who generally needs to understand why a procedure takes the form that it does in order to remember how to apply it, this didn't really work for me.

TEACHING FAIL
posted by grumblebee at 11:48 AM on October 18, 2009 [17 favorites]


The semester I took Intro to Proofs, I and several others in my class became involved with research while we were still in the course. If that's not a low entry barrier, I don't know what is.
The problem with this is that I decided that I didn't like math somewhere around the eighth grade. If students don't find out what's appealing about a subject until college, you're going to lose a lot of people.

I actually really liked high school geometry, because proofs were fun. That didn't change my opinion of math, though. It just made me think that geometry was an exception to the general rule that math was really boring. And I think I actually have perfectly fine natural math aptitude. I just wasn't interested. I have to think that's because of the way math was presented to me, because as a grown-up I realize that math is not innately dull.
posted by craichead at 12:01 PM on October 18, 2009 [2 favorites]


I wish everyone in this thread could have had my wonderful 8th grade math teacher, Mrs. Balliette. Not only did she give me the confidence that eventually got me through Trigonometry in high school with Bs, even though I was an "arts" kid and not a "science" kid, but she helped me understand that math is not so much hard as it is complex, and that it requires focus and yes, gumblebee, problem solving (brilliant insight). She changed my life by showing me how to trust myself and my own ability to understand.

I'm not very good at math anymore, I've forgotten all the concepts, but I've never felt like I hated math, or any other challenging subject, because of her.
posted by nax at 4:05 PM on October 18, 2009 [1 favorite]


I was a student in an elementary teacher education program back in the eighties. After several of my classmates mentioned that they sucked at math and/or hated math I did an informal survey and asked about 40 future teachers how they felt about math, only one said she loved it.

And as a student you can absolutely pick up on that. I don't think I had a teacher who was enthused about math until high school calculus. None of my teachers were excited about science until I hit grade 6.
posted by emeiji at 5:30 PM on October 18, 2009 [1 favorite]


TEACHING FAIL

I think a major thing about math that makes it more intimidating than other subjects is that "teaching fail"s can have such a profound impact. I've had awful literature and history courses but I haven't given up reading. In fact, I could probably pick up any book in the library and analyze it and I'm much further on the science spectrum than the liberal arts. Some people intuitively grok math and aren't harmed by bad teachers. Throughout high school I was like that - but in college my intuitive math ability conked out. Maybe if I had a magical lego music video (or even just a class size below 200) I'd be a math major right now.
posted by fermezporte at 7:48 PM on October 18, 2009 [1 favorite]


I've had awful literature and history courses but I haven't given up reading.

No. But how many people do you know that hate (or are intimidated by) Shakespeare because it was forced on them in school?
posted by grumblebee at 8:07 PM on October 18, 2009


Math education is VERY insular. I started flunking around 9th grade because I didn't understand what it was FOR. It wasn't until I got older and started seeing how Math contributes to music and photography and money and finance and how I could USE IT to SOLVE A PROBLEM that I really "got it".

Math is taught in a really narrow fashion- Math solves Math Problems, not Life Problems
posted by GilloD at 8:36 PM on October 18, 2009


Best answer: I love your job, OP, and I'm fascinated by it. I LOVE what you do, and what your colleagues do. You rock! One of my all-time heroes is Feyman, I've watched as much of him as I can find, though in the lectures that he did at CalTech that Gates put online for us all, the second lecture told it exactly how it is -- if you are not conversant in math, of about every kind that there is, you're not going to make it in the world in which he lived, the world in which you live, OP. And it seems pretty clear that you don't just have to be able to stagger through some math courses but rather be totally and easily conversant in those differing dialects so that when someone spits out lines of thought expressed succinctly and eloquently and fast you can follow along same as if you're talking to the checker in the grocery store about the price of dog food or whatever. I watch and listen, catch what I can; it's like when I was in Paris, and I don't speak French, every now and again there'd be something that I understood "Hey, taxi is same here as at home -- cool" but mostly I was staggering around sortof in the dark as I walked through those wonderful days in the city I love.

Just a great thread, a pleasure to read, gone and come back to it a number of times, cruised the net and come back, fell into a doze I just came out of and finally finished reading it. Great insights. Great stories, and sad ones, also.

We moved when I was in fourth grade. I was 'a good student' and got good grades and all where we'd lived; where we moved to I just never really caught on to much of what was going on, and most especially got totally lost -- and have never gotten caught back up -- in math. Or arithmetic, really -- I'm 54 years old and I can't divide without a calculator.

I got into that new school and they were doing 'The New Math' and it was nothing at all what I'd learned and no one helped me and I flailed and failed, nosed my way through math/arithmetic/whatever grades 4-8 somehow, in high school I'd signed up for Algebra 1 and I never, ever caught any of it, and the instructor -- I won't forget this any day soon -- took me to the hallway and said he'd pass me with a D at the semester break (it was a yearlong class of course) and that'd fill my HS math requirement but the deal was that I could never take another math course. I of course said 'You betcha!' and it was a done deal.

And because I never, ever learned grammer and I didn't then and don't now give a rats ass what a dangling participle is and I never, ever will, I was held in stupid kids English courses and they kept grinding me on that trash until they threw me out of that also. It wasn't until years later that I learned that English classes introduced people to books and writing and I don't know what all; I bet I read more than any other kid in that stinking high school, damn sure as much, but no one ever asked me about that, they asked me about participles, whatever the hell they are.

I got put into all the welcome back kotters kids classes with a bunch of dummies and a bunch of stoners, most of whom were my friends; I majored in drugs for four and a half years. But I'm from a construction family, by my junior year I was making more money than my teachers and I was working a mans work and I knew it and school for me was just a social affair.

Someone upthread mentioned that a carpenter has to problem solve and they are damn sure right. Another day I'll never forget was the day I was thrown into a house with a pile of lumber and a skilsaw and the people were moving in the next day and there were not any stairs leading to the basement. Used to be that when you bought a framing square it came with a booklet not much bigger than a packet of matches that sketchily explained how to cut stairs and rafters; that was my teacher. I'd be hard pressed to cut a rafter today and it'd take me some time to build your stairs and I'd likely hire it out -- now THERE'S some problem solving -- but I can do it.

I'm not dumb. At all. When I trashed my back, mid thirties, and couldn't work construction anymore, the state of Texas voc rehab folks helped me back onto my feet, helped pay for college classes, etc and etc. And they sent me to some sort of testing facility that tested IQ not based upon knowledge of algebra or dangling participles but rather on problem solving logic tests and I was off the charts. I was pretty excited, was ready to do whatever this mope said, if he'd told me I was best suited to be a forest ranger in Ecuador or whatever, I'd have gone and done it, but he told me instead that given my temperament and my intelligence I could pretty much do whatever I wanted to do. Great. Big help that was.

I ended up in a fast-paced program for back-injured people -- three of us where in wheelchairs, Randy had been horribly burned in a fire at a steel mill, Steve had some sort of nerve degeneration going on, Pam was maybe the coolest of us all, she'd bought a basket-case Harley Davidson and had put it back together herself, this beautiful, tall, red-headed, blue-collared gal dumped that Harley drunk one night and ended up in a wheelchair and there with the rest of us, fourteen in all, learning to program mainframe computers. I can tell you that it was quite interesting to learn COBOL without even the rudiments of a math background, lots of the statements are based upon quite simple algebraic formulas that I didn't know, etc and etc. And the text-book we learned from was the single shittiest COBOL text I've ever found, and over the years I looked plenty, to make sure. I was completely and totally lost until I sat down at the keys and began pounding out code and seeing what happened when I ran it.

It was exactly the same as building those stairs. Get thrown in the water and swim or drown. I swam, barely keeping my head above water, while figuring all this jive out.

Turns out I was a pretty good programmer. Check it out -- no math behind me but I was real good at arrays, at keeping the subscripts in my mind, in making sense of them, of being able to follow them down, of being able to trouble-shoot them. I've never worked any deeper than a four dimensional table but believe me, that's pretty dang deep, a row of cubes to be accessed -- Hey! Fun. I interned at Pennzoil, worked for a bank, a state regulatory agency, a bunch of insurance companies, I saved the world from Y2K, etc and etc. Programmed for ten years and change. I didn't like it much but it paid the bills and it was interesting -- how the hell could I be so tired at the end of a day when I'd barely moved in my seat, just staring at a screen and banging out code? Exhausting. Troubleshooting. Problem-solving, I think that was grumblebee -- programming is problem solving, to be sure. And no one but another programmer had any idea what I'd done, which was annoying -- if I'd built out a bunch of walls in a grocery store one day, at the end of that day I could see it, touch it. In code, you don't see anything, except in your head, and in the results of the program when it ran that night, if it did. And if it DIDN'T run that night, then I'm called in and I drive cross-town and to my office and log in and get after it, trying to figure it out, and the entire production run waiting on my getting this thing right. Problem solving? Yeah. You betcha.

While I was a good programmer I was not great, nor ever would have been -- regardless what people upthread have said, some people are born math people, some are born carpenters, some are born painters, some are born writers, whatever -- it's just in you. And in my experience of life thus far, if you're not born to something you can be very good but you'll never be great, never make huge leaps of cognition to the next right step, the clearest path, the simplest and most elegant solution. Just how it is. IMO. YMMV, of course.

Had there been someone in my life to help me -- grumblebee talking about TEACHER FAIL maybe isn't applicable in my case since we'd moved from one school to another and our home was falling apart at the seams and no one there to help me, I was on my own -- but had there been someone to help me I bet I'd have been able to have done the deal. I'd love to be able to learn more about your world, OP, and our world, and our universe -- it's just a great show. I once bought Einsteins book where he wrote to explain relativity for anyone, reduced to it's simplest -- I was excited. But what he meant was anyone who knew the rudiments of math. (sigh)

More. The Indian programmers and later on software engineers I worked with are just on fire with math smarts, they could sit down and talk with you intelligibly any day of the week, of that I am sure. You don't make it into a good school there, in India, unless you're totally dedicated and driven, both by your family and by yourself, to learn and learn it all. And no one gets onto a plane coming here to the states unless they are the best of the best of them from there. We talked it all out; they were amazed by our school system, they literally learn in grade school what Americans learn in college. In fact, they become conversant in it in grade school. Natural selection. Survival of the fittest. Very few schools there, you're not just given a house and an SUV for showing up, you've got to bleed.

Sum: Math is a bitch for many people. Not for all but for many. It's not taught well. It's not 'cool' to be a math geek; he's at home doing quadratic equations and the football players are screwing the cheerleaders. If you can get married and have a big screen tv and an SUV without thinking, why bother? Etc and etc. I'm glad that you're doing what you're doing, I hope you are also.

Peace.
posted by dancestoblue at 10:49 PM on October 18, 2009 [10 favorites]


My dad tried every conceivable way he could think of to explain the concepts to me, pictures included... *showing my age somewhat* if videos existed when I was doing this, he'd have probably tried it! It's because of him that I actually like the stuff I do with it now, and it's because of what he taught me through out all those years of me miserably not getting it that I actually have fairly robust logic game skills now... that I can sit down at the computer and walk him through stuff that he's apprehensive about (pretty soon I get to show him how to set up his router so the whole world can't use it -- me! *I* get to show *my dad* something on the computer! So! Exciting! I know math & computers aren't the same, but the processes of logically working through a math problem & a computer problem are very similar in my brain)...

I see where you're coming from, Grumblebee, & totally respect that you can only see the picture I've painted poorly online, but my dad was (and still is) one of my favorite & best teachers ever. :) I still giggle when I remember him getting annoyed with a particularly bad story problem and saying totally exasperatedly, "why... hrmph... that makes as much sense as asking how far the train goes in furloughs per fortnight!" Solving for furloughs per fortnight (yes, he made us [me & him] work through that too) was a lot more fun than whatever the silly original problem had been!

The things you list, my daughter's teachers do -- they count with real money, use a lot of tactile stuff (blocks, legos, finger paints), they do a lot more creative stuff (track & compare daily weather temperature changes, as one tiny example out of many that they do), breaking stuff down into steps (and this is something that she and I at that age share -- this part is difficult and we miss steps and can't always see where a step needs to be inserted or if we're over complicating something simple). And as she's gotten older, the "fun" tactile stuff has fallen away somewhat & the dreaded state testing & teaching to the test has started up to some degree. We're in Washington, so have to deal with the ridiculous & punitive WASL. We're still at the beginning of this year, but due to the creative work her teachers have done with math (and ok, sometimes *I* don't get what they're trying to do with math, it's so creative) in the years leading up to this year, she's been able to maintain a "student meets grade level expectations" -- so for grading purposes she sufficiently gets it, but its got big scary teeth to her and she doesn't see the fun in it.

Let me be really clear in stating that many teachers can't be this creative -- even if they want to and have the skills. They are hemmed in by all sorts of constraints: financial and ones imposed by school systems and governments. But even if bad teaching is not the teacher's fault, it's still bad teaching. And it's certainly not the student's fault. Just -- yeah. I think this bears repeating for truth because I feel like I'm getting into territory where I want to defend my daughter's teachers. There are institutional issues, time issues, etc. Within these constraints though, recognizing that these constraints limit rather than support good teaching, we've been really fortunate with the teachers my girls have. I'm *so* glad that my daughter has had the good fortune to have the teachers she's had rather than the teachers I had in my early years.

This year she has a teacher who is really passionate about math and science & I'm hoping he has some good tricks up his sleeves. They just had their first test and she was upset by the results of it because she wants to do better, even if it's not fun, she wants to get it. But it's also a lot of frustrating hard work, and trial and error on our part trying to find a way to get to that light bulb moment. We've talked to her teacher, he's pretty much of the opinion that they'll work on stuff & she'll be fine, he's optimistic that she will be able to do what would be expected for this age group. We got to bring the math book home to just keep at home for reference for us too, which is nice since that hasn't been the case until now.

She's taking cello this year and she rolls her eyes when I explain that music is math made into beautiful sounds (it's totally all about ratios & time measurements & fractions &... math!). Ah, being 11... But unlike me at that age, she'll come sit down by me when I'm poking around with the budget stuff I set up in excel, or when I'm charting out my team's monthly metrics, or estimating the time & cost of projects, & I'll explain what I'm doing and how I'm thinking through it. I have hope for her! I don't mean to be saying she'll never get it, just that she'll get it in her own time and way.

It sounds like you were a good teacher, Grumblebee. Thanks for putting in that time & care. As a former student & current parent to students, we really appreciate the good teachers out there. :)
posted by susanbeeswax at 11:46 PM on October 18, 2009 [1 favorite]


BostonTerrier: I dislike math because of the terms "signed numbers" and "integers." Who the hell thought these up, and why?
I dislike math because I resent the fact that it used to be called arithmetic, but now it's called "Math"; short and nasty.


What on earth are you talking about? What difference would it be if we used any other terms to refer to integers? Who the hell thought up the terms "table" and "verse" and why? I also have no idea why you think that mathematics used to be called arithmetic.


Erberus: "2) well-educated individual: he/she is expected to have read Shakespeare, but he/she is not expected to know the two postulates of relativity"

This thought has been around for quite a while; look up C.P. Snow's "The Two Cultures" lecture/book.


More importantly, read Leavis's reply and the surrounding debate.
posted by turkeyphant at 3:41 AM on October 19, 2009


Grumblebee's - I've got a story for you.

So, I've heard from two different sources that my former boss (who is now a director) describes me as the best problem solver he's ever met. I think he's being a little excessive, but I'm willing to admit that looking at a complex situation and trying to figure out what is going on is the part of my skill set I most enjoy using.

So a while back, I came across a presentation on my odd little area of expertise that someone had given at a conference. It addressed a widely recognized problem that no one seems to have a good solution for. Rather than go into a lot of technical hand waving, the person basically said, to make an analogy, is that the reason cars skid is because they're too heavy, and then experimentally demonstrated that cars with added weight didn't skid.

I looked at this for a while an just scratched my head. The author's hypothesis made perfect sense, but the data said exactly the opposite was true (and of course this discrepancy was never addressed - yay deadlines). It was to my brain what a chipped tooth was to your tongue. So I started reading up on the subject, learning the equations that governed it and, ultimately, building a big ugly model of the situation on the computer. What I found was that the model people (myself included) kind of carried around in their head had one moving part, but even in my horribly oversimplified model (with hundreds of moving parts) the observed behavior was totally predictable.

So the end of the year comes and I'm getting feedback from various sources - the big criticism? I'm too theoretical, which might be fine for academia, but in industry blah blah blah....

I was gobsmacked. Here, all these years I'd been thinking that knowing what the hell I was doing was a good idea, only to find out that I was piddling away a promising medical career! (Because I kind of know where the appendix is located. Sorta.)

I think this is the mirror image of the issue brought up in Shop Class as Soulcraft. For some reason people want this compartmentalization of doing and thinking, which is totally at odds with the reality of doing or thinking.
posted by Kid Charlemagne at 5:51 AM on October 19, 2009 [4 favorites]


I know no one is reading by now, but I had to come in here and stick up for a group of people I've rarely seen so maligned and abused on MetaFilter. A group of dedicated individuals who work their asses off in the face of little to no support from everyone around them, and who are constantly being undermined by the people they are working so hard to support. Yes, I am a high school math teacher and I need to set some things straight.

Let me begin by saying that I have degrees in Engineering Chemistry and History. I've seen "both sides" of academia, and I've been a woman in math and science my whole life. I'm tired of people blathering on about how the math world just isn't woman-friendly. Bullshit. We spend far too much time talking about women's difficulties in various fields theoretically, and far too little time just getting in there and doing whatever it is we want to do. My female paramedic and pilot friends agree. If you look hard enough for bias, you'll often end up creating it all on your own. But I digress. Let me get back to the point, which is defending my fellow educators.

Let's start with the very beginning of a child's life, and their introduction to the process of education. ABC's all over the place. "Dog", "horse", cats in red hats, etc. Any mention of numbers, glorious numbers? Some integer lines to go with those alphabets? The introduction of our powerful friends Ms. Multiply and Mr. Addition? No. Never. Counting on the fingers is about as far as we get. By the time kids get to school, they may well have a great grasp of letters and how they come together to make words, but they have no notion of the power of zero and the orderly left to right progression of negative to positive, or of the difference between an even and an odd number. I think it's disgraceful. I have a hand-painted and laminated integer line marching around the top of Go Banana Jr's bedroom. Hand painted you say? Yes, because no such product exists and I had to make it myself (though there were alphabet posters to spare). From the very start of our children's existence, they are taught that numbers are mysterious and unknowable. They are not made accessible. This is a huge problem that I, as a high school math teacher, struggle against every day.

Sadly, the elementary curriculum (note: I'm talking about Ontario here) does nothing to address this injustice. Everyone seems to think said curriculum is too focussed on rote learning and memorizing of times tables. Ha!! I wish!!! Math in the elementary grades is all about problem solving and social justice, with wanton use of calculators and a liberal sprinkling of Individual Education Plans that dictate that little Johnny doesn't have to memorize his times tables because he has anxiety. I appreciate what they're trying to do, but kids need to learn their damn times tables. They really do. Kids already see numbers as a great unknown. Leave the problem solving until later and focus on getting to know those numbers. Get intimate and personal with them. Study their interrelationships, learn long division, play games with them and enjoy their simple power and majesty. We need to teach children to love numbers and be comfortable with them. In order to do that, they need to be able to do all those "boring" things like add, multiply, subtract and divide. Without a calculator. Calculators just feed the fear of numbers at that level. There are two big problems in elementary math, however. The first is the curriculum. The second is the teachers, who usually don't share my love of numbers and tend to give short shrift to them in a full day of learning. We need to teach our elementary teachers to love numbers so they can pass that on to the children.

This is where the high school math teacher comes in. We follow an early childhood devoid of number power, and years of elementary school too focussed on higher order thinking, and not enough on basic skills. And we're supposed to teach them algebra, statistics, and advanced problem solving. So we try. We try to teach them to factor quadratic equations so they can solve delightful word problems about Wile E. Coyote shooting a cannonball at that pesky roadrunner (because we really try to make this stuff fun, honestly we do). I'll tell my class of 30 fifteen year olds, already convinced that "math sucks", that to factor the equation describing the path of the cannonball through the air, we need two numbers that add to 10 and multiply to 24. And they're totally stumped. "But Miss, how do you know what the numbers are?". So we go though it. List the factors of 24, and find the two that add to 10. If they don't know their times tables (which they usually don't), or if they are reduced to trial and error on their calculator, the whole process becomes so tedious they've lost the fun entirely. They keep loudly believing that math sucks, is totally useless, that they suck at math, that I am an evil ogre out to get them. Can you see why so many math teachers just give up and become boring automatons? There are profoundly useful math courses in high school too, like Math for Everyday Life, which teaches kids how to do their income tax, lease a car, and keep a household budget. Great stuff, but we struggle to get kids in the class because the mere mention of the word "math" is enough to send them screaming.

I guess what I'm trying to say is that we can't blame society's attitude towards math on teachers. We never even stand a chance by the time we get your kids in many cases (though we do get through to some of them, and that makes the whole thing worthwhile). We need to pay more attention to the example we set, and our own attitude towards math ("Oh, I don't do my own taxes. Who can understand all that?"). We also need to teach our kids their numbers along with their ABCs. And we need to give our kids the basic math skills they need. The math curriculum now is too often the equivalent of trying to teach someone who is illiterate how to write an essay. It's hopeless, and it's doing nobody any favours. No one would try to say that kids don't need to memorize their letters and learn how to use punctuation. So why do so many people continue to assert that kids don't need to memorize their times tables and learn how to use mathematical symbols?

So please, MetaFilter. Give math some love, and spread that love around. Maybe save a little for the math teachers too, because they could really use some support.
posted by Go Banana at 8:09 AM on October 19, 2009 [11 favorites]


I've always thought maybe I wasn't particularly good at math because I've been unconciously assigning gender, psychological traits and behavioral characteristics to numbers for as long as I can remember. 7, for example, was male, sort of domineering, self-righteous (and also purple). Although his heart was in the right place (and many people mistook him for a savior), he regularly made these epic mistakes, that often imperilled his less-confident, older brother, 5, and complicated his already complicated relationship with the lovely, but naive 12.

I'm slightly ashamed to tell you I spent most of my academic career struggling to not let my brain go there. ("No, you can't subtract 48 from 116, because then you'll end up with 68. And 68 is very sweet and maternal on the surface, but I have reason to believe she's in cahoots with 17 and 17 is a real bastard. Remember that one time, he tried to impersonate 71?") Or ("Jesus, you're going to let 3 go all exponential, do you have a death wish or something?). When I first learned about negative numbers, I just figured they were the antithetical doppelganders of their positives. -7, for example was a self-conscious, indecisive girl, who acted out of vindictiveness, but usually ended up doing the right thing in spite of herself. I think one of the reasons why algebra derailed me was that I found myself assigning personalities to letters and numbers and was often disappointed when the character of the letters didn't neatly line up with the character of the numbers ("N" can't just stand in for anything. Not with the creepy way he has of being silent from time to time and his constant, threatening harassment of the letter "o").
posted by thivaia at 8:34 AM on October 19, 2009 [7 favorites]


I think the reason people dislike problem solving (as grumblebee eloquently explained to much acclaim) is that people are overly afraid of being wrong. I'm a programmer, and I hate being wrong, but I'm not mortified by it.

I get things wrong all day long, but when I find out I'm wrong about something, I don't curl up and die and think, "Oh, man, I suck, and everyone else is figuring things out on their first try!" I do sometimes get frustrated that I need to spend more time on a problem (although some attempts at solving a problem are very quick and inexpensive), but I do value finding out that an approach to a problem is wrong for some reason. I can use that information for another approach to that problem and perhaps other problems.

I think that this does come partly from school systems often not giving math students credit for the thinking they did to arrive at their wrong answer*. The rest of it is from our culture often punishing people that try and fail at something publicly and leaving alone people that never try anything and therefore fail at nothing.

* If you are going to only give absolute credit, though, a bigger emphasis on showing students how they can check their answers themselves would go a long way. This is possible with a lot of algebra and geometry, and even with arithmetic.

My third grade teacher, Mrs. Tomick, showed me a pretty neat trick in which you can check to see whether or not you multiplied something correctly like this:

So, say you multiply two numbers and come up with this:
    49
x   15
------
   245
   49
------
   735
To see if this is right, you can replace each of the 9s in the factors with 0s, then add them together:
49 => 49 => 4 + 0 = 4
15 => 1 + 5 = 6
Then, multiply those numbers together:
4 x 6 = 24
Then, add those numbers together (replacing 9s with 0s as before) :
4 x 6 = 24
And add the numerals of those results together:
2 + 4 = 6
Now, do the same (convert 9 to 0, add numerals together until it comes down to a single digit) for the result:
735 => 7 + 3 + 5 = 15 => 1 + 5 => 6
So, the numbers match! You know that your original multiplication was probably correct. I think you can get false positives this way (e.g. if you came up with 6000, this method of verification wouldn't say you're wrong), but if you came up with something like 635 by not carrying a 1, it would tell you you did it wrong.

I am definitely not a math genius, but this was a tool that gave me a lot of confidence in my multiplication and division abilities back in the day. I didn't have to wait to see if I did a problem correctly. It the power into my hands, and made me see that it's not the end of the world to get something wrong and that mistakes could be fixed.

I think that throughout arithmetic, algebra, geometry, and calculus, there are self-verification mechanisms that can take the emphasis off of worrying about whether or not you got the problem right on one particular attempt and focus the student on the process of solving the problem. If nothing, you can always hand out an answer key and make everyone show their work or talk through their solutions.
posted by ignignokt at 8:50 AM on October 19, 2009 [4 favorites]


They are given class deadlines and then poor grades, and later moved forward without mastery. This slowly causes anxiety. Anxiety equates to fear and loathing.

I'm terribly late and I know no one is reading anymore, but I wanted to comment on this and share my reasons for why math is "scary" (maybe typing it out will be therapeutic!).

The first time I can recall having difficulty in math was in third grade, learning multiplication. My teacher would explain it, then give us worksheets to practice on. I sat there and stared at my blank sheet. I didn't get it. I'd go up to her desk, she'd explain it again, I'd experience a momentary, fleeting understanding, and I'd go sit back down thinking I could surely do the rest on my own. And poof, it was gone as soon as I got back to my seat. I'd go back up to her desk, ask her to explain the next problem, and the same thing happened. And this experience repeated itself pretty much every single time I did any sort of math for the next eleven years. With a boatload of tutoring and begging for extra credit, I could scrape a decent grade to pass to the next level. But I never understood it on my own, could not have explained it if you paid me, and progressing each year to more complicated math without having mastered the previous year's lessons was extremely stressful and anxiety-making.

I sort of understood it when someone else was explaining it, but that understanding was gone as soon as I tried to replicate it on my own. It's hard to explain, but trying to do math feels like walking in an extremely thick fog that has taken over my brain. I'm disoriented, I can't see, I can't think. It's like static has taken over my gray matter and I can't cut through it. I wouldn't call this "scary", exactly, but it is intensely frustrating and upsetting. Even reading your question upset me because I feel like there is no way I could make you—or anyone—understand how far out of reach it feels. My dad is very mathematically-inclined, and he used to "help" me with my math homework every night, which was excruciating. We'd sit at the dining room table for hours in the evening. I quit asking for his help in tenth grade after one night when he flung my textbook across the room in a rage because I hadn't read the explanatory pages at the beginning of the lesson. I hadn't read them because there was no point, because I'd found long ago that they didn't make sense. Reading them would not have helped.

Yeah, sure, better teachers might have helped—I have only ever had one math teacher who didn't act like my constant requests for help were a huge burden, and I will always be grateful to him. As would more practical applications—at the risk of sounding mind-blowingly obtuse, a comment upthread about throwing a ball in the air is an example of the Laws of Motion caused a light bulb to go on over my head. I've never thought of it like that. I'm not sure I know how to think like that.

I know how this sounds. If anyone else came to me and told me about the static brain phenomenon, I'd have rolled my eyes and told them that maybe they just need to focus harder. But it is real, and it feels like a math handicap, or a learning disability, or something. I want to understand math, want to master it, and I hate that I can't seem to (example: I've been studying ignignokt's trick in the post above for about twenty minutes now, am still only halfway through, and it's way more confusing to me than just doing the regular multiplication). To do that, I'd have to go back and reteach myself everything that comes after basic division. I wouldn't know where to begin to accomplish that, and I don't have a huge incentive to do so, because I've chosen a path in life where it isn't all that relevant.

I guess this is a long-winded way of echoing "people have different brains".
posted by anderjen at 2:29 PM on October 19, 2009 [1 favorite]


anderjen, maybe you have a learning disability with math; maybe you don't. We DO know it's a hard subject for you.

What you needed was a grownup who said this, "I don't know if I can teach you math, but I can promise you this: I will enjoy helping you and working with you for as long as you're willing to work with me. I will NEVER get mad at you; I will never call you stupid; and I encourage you to ask me the same questions over and over if you need to..." And that grownup would have needed to keep that promise.

AND he would have needed to be the FIRST grownup who taught you math.

If the first one is not like this -- if the first one is impatient with you -- then math, a subject you already find difficult, will be connected in your mind with being chastised or laughed at. And that will make the second teacher, even if he's good, have a really hard time getting you to trust him.

If even a small part of you is worried about how the teacher is going to react, you're going to then have a major learning disability -- whether you had one before or not.
posted by grumblebee at 3:26 PM on October 19, 2009 [2 favorites]


a great thread, and I'll have to come back later this evening for a closer read.

I suspect that math is the single most difficult subject to really teach. History can be taught as "just one damned thing after another" with some degree of success, but not math. Math requires as much creativity as any English composition class, but the result of the process is usually (and correctly) either right or wrong. It's no mystery why students find this frustrating.

So, to all metafilter math teachers out there: keep up the good and clearly very difficult work!
posted by lex mercatoria at 4:03 PM on October 19, 2009


"Unfortunately, creativity is not required. To reach the level where creativity is required, I am afraid that you'll need to do a PhD in maths."

This is absolutely incorrect. I'll grant that creativity is not (always) required to get a BA or BS in math -- it is probably possible to skate through some or most math programs by regurgitating what the professor said. But you definitely don't need a PhD (or any other special certificate) to do creative mathematics. There are plenty of problems that require creative solutions that can be understood by someone with no more than a decent understanding of basic algebra or geometry. For instance, proving the Fundamental Theorem of Algebra takes some mad cleverness, but both the problem and proof can be explained to high-schooler. The semester I took Intro to Proofs, I and several others in my class became involved with research while we were still in the course. If that's not a low entry barrier, I don't know what is.

Yes you are correct, I'm afraid I didn't explain very well. I should rephrase: creativity is probably not needed for the 1A, 1B maths for natural scientists tripos and 1A and IB maths tripos here in Cambridge. Although I believe this is also true for many of the other maths courses in the UK (hearing from friends in other unis), I do admit other courses in other universities may differ (especially not in the UK). Essentially, I was speaking out of experience. Unfortunately, my uni doesn't offer any incentive to be creative in maths :(
posted by dragontail at 5:12 PM on October 19, 2009


Best answer: I'm now so far down the long tail of this thread, I doubt anyone is still reading. But I want to talk about one more issue, one that we've been skirting around, which I think pertains directly to the O.P.'s question: I want to talk about math as an aesthetic object.

There are plenty of practical reasons to do math: you need math to build bridges, launch missiles and code video games. Such math will be of great interest to bridge builders, missile launchers and video-game developers. No one will have to push them to learn math. They'll learn it because they need it.

Most of the rest of us don't need math to get by. Or, rather, the math we need to get by could be taught to us in a week: checkbook balancing, tip calculating, etc. So students are quite sane to ask "How is this relevant to my life?" And I've never heard a teacher give a good and honest answer to that question.

The closest I've heard is from teachers who rail against innumerate voters. How can we expect democracy to work when people can't tell the difference between ten-thousand refugees and a hundred-thousand refuges? How can we move safely and productively into the future when we don't have enough scientists and problem-solvers?

Sad as it is, though, this is not what people mean when they ask how math is relevant to their lives. They mean how often will they need to use math when they are taking out the garbage, making soup in the microwave, riding the bus to work and playing with their kids? In short, they don't want to know how them being good at math will benefit society; they want to know how it will benefit them, right now, or in the foreseeable future.

The honest answer is "almost never."

I think it's time to for educators to face the fact that people mostly don't need math. Neither do they need Shakespeare, Keats, Picasso or Mozart. Why learn to appreciate Mozart? Because your life will be aesthetically richer if you do! Why learn Calculus? Because, like "King Lear" or "The Magic Flute," it's one of the great achievements of human thought. And you'll be in touch with what's divine in humanity if you learn it!

Many of us who are cultured want to live in a cultured world. We want people around us who are cultured. To some of us, that means we want to be around people who know history or read poems. To others, it means they want to be around people who know Set Theory. There are cultural traditions that we want to be part of -- and we want to pass those traditions onto our children. And the passing on of traditions has always been (and will always be) fraught. For every little boy who loves going to Sunday School, there are a dozen who would rather stay home and play video games.

The ethical and practical issues surrounding this are so befuddling that most of us refuse to confront them. In a nutshell, they come down to this: do we have a right to force kids to read Shakespeare or do Algebra problems? If so, how do we defend that right? I don't really think there is a defense for it. I think the truth is that we who love Shakespeare and Algebra are selfish. We want other people to be like us. I very much count myself in this group.

Yet when a kid says, "Mr. Math Teacher, why do we have to learn how to graph functions?", how many Mr. Math Teachers are going to be honest enough (or have thought through it enough) to say, "You won't, but it's a beautiful thing to me and so I want you to learn it"? Most teachers lie or parrot bullshit that they don't really think hard about. Something about how much you'll need to graph functions later in life. Something about training the brain. Something about America needing more mathematicians. (And the problem is, kids have great bullshit detectors. Bullshitting a kid about why you're forcing him to do math is yet another way to make him hate math. He'll associate the subject with the bullshit.)

We desperately need to have a discussion about aesthetics and education. And it won't be an easy discussion. There are some really tough questions, such as whether or not it's okay to force people into having aesthetic experiences. Is it better to require people to listen to Mozart or to just hope they stumble onto his music by themselves. Does it even work to force Mozart on people? If people are forced, don't they generally wind up resenting what they are forced to do?

Is there some way to not force but to create environments in which many people will find their ways to Mozart and Newton on their own?

I should add that some aesthetic experiences are relatively easy to foist on people. Food, for instance. It doesn't take hard work to turn someone into a foodie. Food is instantly accessible.

The tough aesthetic I'm most familiar with is Shakespeare. You can't really appreciate Shakespeare without doing some hard work. Sure, you can go see his plays and chuckle at some of the jokes you manage to get -- or be thrilled by an actor's performance. But that's not REALLY getting Shakespeare. I always know I've run into someone who doesn't truly appreciate Shakespeare when he says, "You know, Shakespeare wasn't meant to be read." Yes, it's great to see "Macbeth" on stage, but people who have REALLY fallen in love with Shakespeare carry a copy of the play in their pocket. Of COURSE it's meant to be read. It's inspired poetry!

After reading SLOOOOWLY though several plays, looking up each word I didn't understand, and basically learning a foreign language (Elizabethan English), I "got" Shakespeare. And WOW what an EXPERIENCE! It was so thrilling, I wanted to share it with everybody. And it seemed so BASIC, so much about what it means to be human, so IMPORTANT, that I felt like all school children should be required to do the work that I did, so that they too could touch the feet of God.

Trouble is, it requires all that work. If you don't speak Elizabethan, you can't just pick up a copy of "Othello" and instantly go into spiritual rapture. What is going to keep you plugging away before you have that experience? You have to take it on trust, based on reports of other people, that if you work hard, you will have that experience.

I have given a lot of thought to the minimum amount of work one would need to do to have such an experience. In other words, if I taught a one-week Shakespeare class in which I focused on just one scene from "Romeo and Juliet," could I bring the students to rapture by the end of that week? I don't know. I haven't had the opportunity to try.

Math teachers should give this SERIOUS thought. The longer it takes to do the basic work -- the work you have to do before math becomes transcendent -- the fewer people will be willing to do that work. SOME work (much work, I'm sure) must be done, but what's the minimum? Is there some way we could bring kids there -- even momentarily -- after a few months of Algebra I? Because if kids just had a tiny dose of what math CAN be to the soul, they might yearn for more. I'm guessing many readers here can't even begin to fathom how math could be beautiful or spiritual. We need to give people that aesthetic shock as soon as possible or we'll lose them.

Should we do this? My gut tells me that even if we should (and I'm not at all comfortable with the idea of forcing aesthetic experiences on people), it won't work most of the time. Force kids to study Shakespeare in a Talmudic way and you'll get a lot of Shakespeare haters. Same with math.

So this brings us back to the key questions: do we require aesthetic experiences? If so, HOW do we bring people to have aesthetic experiences? If not, are we okay with letting people go to their graves missing out on the sublime? Is it okay for people to have some aesthetic experiences but not others? (How do we feel about someone who comes to love poetry but hate music?)

Bringing this full circle to the O.P., he HAS touched the feet of God in his discipline. He knows how math can transport the soul. He is always going to have trouble talking to people who haven't been in his shoes because he is, essentially, a Christian preaching to a group of atheists.
posted by grumblebee at 6:44 PM on October 19, 2009 [22 favorites]


I love this thread. I'm *so* with you anderjen -- I know that thick fog. And BananaGo & grumblebee, I love your insights, thank you for sharing them.
posted by susanbeeswax at 10:28 PM on October 19, 2009


math contains no love or emotion

e^(pi*sqrt(-1)) = -1

If that doesn't make you feel something, either you don't understand the notation or you're dead inside.
posted by jewzilla at 1:25 AM on October 20, 2009 [4 favorites]


I sort of understood it when someone else was explaining it, but that understanding was gone as soon as I tried to replicate it on my own. It's hard to explain, but trying to do math feels like walking in an extremely thick fog that has taken over my brain.

This happens to me all the time, but I know how to get past it. My method wouldn't have worked in school, because it only works if I take my time, and it might not work for anyone else. Then again, it might.

I often have to (or want to) learn things that are just about, but not quite, beyond my understanding. I have that glimmer of semi-understanding and then the fog descends. I feel the subject getting further and further away and it's hard to see it through the murk. And I feel exhausted. My brain hurts. It feels as if it's used its last calorie just trying to hold on.

Here's what I've discovered: that glimmer -- even if it only lasts for a second -- means that I AM capable of getting it. The fog and fatigue is my brain telling me, "DON'T PUSH IT! You have a chance of getting this in the future, but if you keep trying right now, you'll lose it forever. You NEED a rest."

So I shelve the subject. Sometimes I need to shelve it for a month or even a year. Then I come back to it. I try to come back to it from another angle -- a completely different book or teacher. Sometimes I have the same glimmer-then-fog experience. Sometimes the glimmer lasts a little longer this time. Sometimes, if I'm lucky, there's no fog this time.

If I keep trying this technique -- taking a break (sometimes a LONG one) when the fog descends -- I WILL get the subject in the end. If I don't take a break, I won't.

In school, you can't take a break. The fog descends, but you still have all those problems to do. And you're not allowed to raise your hand and say, "Teacher, my brain is telling me to stop for a month!" Which is too bad, because via this method, I've learned all sorts of things I never thought I could learn. It's too bad that the school method reinforces the fog rather than the subject. Trying to push past the fog makes the fog thicker and thicker and thicker. Eventually, it turns into a brick wall.

I suspect that many people don't keep trying, because once they get out of school, they are relieved that they don't have to. If they feel the fog, they drop the subject for good. "It's not for me," they say. Or, by the time they're grown up, they know enough about themselves to know what subjects will bring on the fog. So they avoid those subjects altogether. This is too bad, too.
posted by grumblebee at 7:43 AM on October 20, 2009 [4 favorites]


Possibly a proximate cause is the evisceration and destruction of JHS and HS shop classes. If a student never uses the math he or she are learning outside of math classes, some won't bother to learn it at all. This is exacerbated by weak science teaching.

One response is the FIRST Robotics and other competitions. Along with HS robot clubs, etc. etc..

Another, computer-enabled, response seems to be the Fab Lab movement.
posted by sebastienbailard at 1:19 PM on October 20, 2009


I think it's like, when you consider posting a comment on Metafilter, but don't, because you know people will tear it apart. You know the feeling. I am guessing that is what math is like for a lot of people.
posted by water bear at 10:21 PM on October 20, 2009


I would like to offer a different (I assume because I haven't read all of this post!) possibility:

Math is unpopular because it generally needs a two-step argument at some point to become useful.

This is similar to the "problem solving" answer, which I agree with, but there are some "problems" which you can solve without taking logical steps and producing an argument.

In the mediaeval "trivium" model of learning, you must pass through the 3 stages of grammar (memorisation), logic (argumentation) and rhetoric (relevance to audience).

Far from being arbitrary, this is actually an empirical description of learning and communication. If you look at a modern textbook on this method, you will see striking similarities to processes we use in various fields under different names (e.g. Boolean logic).

Very rarely in our modern society do we explicitly use the whole 3 steps, and so people are not used to it.

Math and mathematical sciences are more scary simply because it is harder to get the answers without passing through those stages more explicitly. It's easier to cover ones' tracks, so to speak, in other fields (because of how our society is set up).

They (sorry, we!) just aren't used to it, simple!
posted by KMH at 4:03 AM on October 21, 2009 [2 favorites]


Also on the same lines, as Sayers says in her speech that I linked to (and as many others have debated here!) the academic subjects are taught to us often as though in watertight compartments and never mix with each other or with our direct experience.

Again, a mediaeval may have been factually rather, ahem, lax - but methodogically it was ideally very vibrant and "interdisciplinary".
posted by KMH at 5:20 AM on October 21, 2009


Yet when a kid says, "Mr. Math Teacher, why do we have to learn how to graph functions?", how many Mr. Math Teachers are going to be honest enough (or have thought through it enough) to say, "You won't, but it's a beautiful thing to me and so I want you to learn it"? Most teachers lie or parrot bullshit that they don't really think hard about.

A truer response to the question would be: "You don't need to learn how to graph functions, but the people in charge of funding this program are only interested in finding out who can and who can't."
posted by Brian B. at 9:37 PM on October 21, 2009


Way late to the party, but can't resist.

I'm a third year high school math teacher in the US and I love my job so much I want to make out with it (my job, not my students - they are awkward and zitty and smell like feet). If you had told my high school self that I would grow up to teach math, I would have laughed my ass off at you. I hated math. Loathed it. I could do it well enough, sure, but it felt like a complete waste of time to me. Who cares about manipulating numbers and memorizing formulas if you never get to see the big picture, if you're never taught where those formulas come from or how they can be applied?

I could rant for hours about how broken our public education system is and how fucked up our cultural attitudes about math are, but that's territory already pretty well-covered on this thread. What I'd really like to do is give a shout out to all the math teachers out there who actually really dig what they're doing and are able to turn their kids on to it (there actually are some of us out there, in spite of what the anecdotal evidence on this thread might lead you to believe) and also to suggest that anyone who likes math can make a difference simply by being vocal about why they like it and, more importantly, why it's useful. And I'm not talking about the bullshit reasons adults usually give kids, like, "you'll need to calculate a tip" (big deal) or "you'll need to do your taxes" (which most people just outsource anyway). I'm talking about "we have an impending worldwide food and water shortage and need the best minds available to work on solving the problem" or "the Hubble space telescope beams back massive quantities of data and we need people who can sort through it and make sense of it" or, even better, "marketers and politicians will try to manipulate you by citing bad statistics and you need to be clever enough to know which imformation should be believed and which can be disregarded so that you won't be taken advantage of."

It's true that not everything I teach is useful. Some of what I teach I teach because the state or federal government mandates it. Some of what I teach I teach because it's of historical interest. Some of what I teach I teach because it's beautiful (yes, beautiful). Transparency in teaching goes a long way. Most kids will buy it if you tell them why you're selling it and if you make it look good enough to at least taste a little of. Are there shitty math teachers out there? Hell yeah. Instead of dwelling on that, though, go find a kid who's got one of those shitty teachers and spend some time talking to them about why math is worth learning in spite of their negative experiences so far. At its core, math is about problem solving, and good problem solving skills benefit you no matter what you do in life. The "I hated math, too" conversation just reinforces for kids that it's ok for them to hate math.

And that's bullshit, because math fucking rocks.
posted by tits mcgee at 6:57 PM on October 22, 2009 [6 favorites]


Guess I learned math old school. Learned the tables, not a lot of fluff in the stuff I learned. We didn't get math for home budgeting, or math with Wile E. Coyote, heaven forbid. I cannot imagine the straight-laced math teachers I had teaching that kind of course. It's also hard to imagine kids get through school without having to memorize the basic multiplication table.
The only regret I have about math was not learning its application in the real world. How sine waves predict all kinds of behavior of real things for example. It was all well and good to learn this stuff but without using it or seeing it as useful, it's all washed out of my head with time and disuse.
One stellar moment in my math life. In high school a physics teacher asked us to calculate the gravitational constant in another universe. Instantly, years of SF, comic book and movie consumption lit up in my head. Wow! I was so into it I flew through the calculation like a rocket scientist.
Perhaps they do that now, but then it was all math with few applications.
posted by diode at 9:55 PM on October 22, 2009


marketers and politicians will try to manipulate you by citing bad statistics and you need to be clever enough to know which imformation should be believed and which can be disregarded so that you won't be taken advantage of.

YES. I gnash my teeth every time I see this HP printer commercial, in which "65% more" is depicted as something like 400% more. LIES LIES LIES. This should not even be LEGAL.
posted by emeiji at 12:54 PM on October 23, 2009 [1 favorite]


Against School: http://www.wesjones.com/gatto1.htm (Thanks oflinkey.)
posted by grumblebee at 10:52 AM on October 24, 2009


The way it (was) taught in U.S. elementary and middle schools has a lot do with it, I think. Imagine if art classes all through elementary school were all about rote practicing of certain types of lines and shapes, rather then a "fun time" activity for students. A lot of people would hate it.

Math was taught as if the kids were going to go into 1950s jobs where they had to do arithmetic by hand all day. It's absurd. All the elementary arithmetic taught to students is obsolete and a huge waste of time. Teaching kids mental math would be a great thing to do, and I think some school districts do this now. Teaching kids to do base-10 arithmetic on pencil and paper over and over again just makes them hate math.

Teaching kids how to use a calculator, some basic mental math and then moving on to advanced topics as well as giving people an overview of all the different kinds of mathematics out there (I never learned anything about combinatorics or vector spaces in elementary/middle/highschool) would probably make people like it more.
posted by delmoi at 10:46 AM on October 29, 2009



e^(pi*sqrt(-1)) = -1

If that doesn't make you feel something, either you don't understand the notation or you're dead inside.
If you actually understand why that is, it's not really that interesting. Any time you raise e to an imaginary number, you end up involving trigonometric functions. here's an explanation
posted by delmoi at 10:59 AM on October 29, 2009


BostonTerrier: Math never used to be arithmetic. Learning arithmetic is just learning how to do basic operations on numbers like addition and multiplication.

Try reading Jean Pierre Serre's A course in Arithmetic. From one of the user reviews:
Serre's goal in this section is to give a complete classification of the quadratic forms over the rationals. As preliminaries to reaching this goal, he introduces the reader to quadratic reciprocity, p-adic fields and the Hilbert Symbol. After these three, he spends the next chapter detailing the properties of quadratic forms over Q and Q_p (the p-adic field). The reason to work over Q_p is the Hasse-Minkowski Theorem (which says that if you have a quadratic form, it has solutions in Q if and only if it has solutions in Q_p). Using Hensels Lemma, checking for solutions in Q_p is (almost) as easy as checking for solutions in Z/pZ. After doing that, he spends yet another chapter talking about the quadratic forms over the integers. (Note: the classification goal is already achieved in previous chapter).
posted by delmoi at 11:11 AM on October 29, 2009 [1 favorite]


I just finished reading the whole thread. Really interesting stuff. I was always bad at arithmetic in elementary school. The thing is, these days arithmetic on paper isn't really very important at all. I've never really used it. In highschool I actually had a calculator that could do algebra and even basic calculus. The fact that I had it meant I could concentrate on the concepts and ideas, and not get too tripped up so much by making little errors as I went along.

I'll share a "bad math teacher" experience. I actually got yelled at and told I was "So slow" at one thing. I think it was long division. I remember I always kind of blew off long division in earlier grades. I figured I would never need it. And you know what? I was right.
posted by delmoi at 12:20 PM on October 29, 2009


A truer response to the question would be: "You don't need to learn how to graph functions, but the people in charge of funding this program are only interested in finding out who can and who can't."

If you can't graph a function, and you can't read a plot, and you don't understand how to graphically relate two variables, then fat chance being a pilot, or pharmacist, or doctor, or GAH! lot's of things.

It's kind of sad, really.
posted by sebastienbailard at 3:43 PM on October 29, 2009


If you can't graph a function, and you can't read a plot

Huh? What are you talking about? To be honest, it never even occurred to me that people once spent time in school learning how to graph plots. For me it was 5 minutes one day in Algebra II where the teacher explained how to enter equations into the "y=" screen on our (school provided) graphing calculators.

Learning to graph functions by hand today would be like learning to how to use a slide-rule, or abacus. Now obviously you need to know how to read them.
posted by delmoi at 3:55 PM on October 29, 2009


Some cranky High School chemistry teacher summoned a meeting with my father, to explain why he was failing me. My test scores were all good, but I wasn't big on homework. Most teachers would usually let that slide if I compensated with vigorous classroom participation and elective special credit projects.

Not this old guy. He wanted that homework. "This kid, this kid could DO this homework! I mean, Christ!, the kid gets up to the blackboard and balances an unbalanced equation! And he can't do homework?" My Father ruefully shook his head. I'd never felt smarter in my life.
posted by StickyCarpet at 8:56 PM on October 29, 2009 [1 favorite]


it never even occurred to me that people once spent time in school learning how to graph plots

The closest I ever felt to math was looking at an equation, scoping for zero crossings, accretions and diminutions, accelerations and relaxations. Then, with a rough intuitive shape in mind, choosing just the least points around inflections to plot by hand to reveal a portrait of the described subject.

I knew I was going on to Art School, so math was purely for amusement. Graphing plots by hand was like life drawing class, typing equations into plotting machines was more like watching TV.
posted by StickyCarpet at 9:09 PM on October 29, 2009


You'll might read my postdoc advisors blog.
posted by jeffburdges at 6:48 AM on October 30, 2009


This thread is now being discussed on reddit.
posted by grumblebee at 8:26 AM on October 30, 2009 [1 favorite]


I was talking to a friend about Russian university examinations, and the understanding I walked away from was that many undergrad exams have the student working things out on the blackboard in front of an instructor or two.

Having a student work out problems on a blackboard seems like the best way to evaluate, extend, and motivate their understanding, as opposed to:

1) Skim for an answer that looks right (or work out the problem if you have time/interest).
2) Fill in little bubble.
3) Have robot grade exam.
4) Look up score on computer.

There's an immediacy to the testing on blackboard model, where the computer testing model distances and disengages the student from his or her results. There are some students who will choke up on being tested this way, but you can sort that out with supplementary paper-based exams.

(Oral exams aren't something novel, but I think they've been neglected.)
posted by sebastienbailard at 12:59 PM on October 30, 2009


Not this old guy. He wanted that homework. "This kid, this kid could DO this homework! I mean, Christ!, the kid gets up to the blackboard and balances an unbalanced equation! And he can't do homework?" My Father ruefully shook his head. I'd never felt smarter in my life.

I hate this attitude. It really doesn't matter that much whether you can understand 10th grade chemistry on the first try. Maybe that makes you "smart" by some trivial definition; whoopty-shit. It's not some kind of great achievement to balance a simple reaction; it's no virtue to be able to do it fast, or in your head, even. What matters more is what you do when you come on a problem you don't know how to solve immediately: do you attack it and beat and claw at it until you've found the answer, or do you just throw up your hands and go back to stuff you can do without having to think? Because if it's the latter, you are absolutely no better than the people who don't take chemistry because it's "too hard."

A good problem solver will always be a better chemist than some lazy smartass who can do a few tricks with entry level material. But, as grumblebee has pointed out so well, schools don't teach problem solving; our culture does not in general value problem solving. Instead, the kids who can ace every exam without trying are deemed the "smart" ones, the ones who "get it", and everyone else is just "dumb" or "not a math/science person."

Don't get me wrong, I was a lazy smartass, too: I rarely had a problem understanding things, but I was (in the words of one of my least favorite teachers) "allergic to homework." To this day, it is my fervent belief that the very, very great majority of high school work is a complete waste of time, and I never hid this opinion from my teachers. But I was also very fortunate in that I had a few different opportunities to do some actual problem solving, the two most notable being the stage crew and FIRST Robotics; both of these activities tended to conflict with school, and I always put school second, for which I have no regrets. The shop was where my real education was. I learned how to solve new and different problems, how to solve familiar problems in creative ways, how to bounce back after a defeat, and most importantly, how to work steadily at something until it was done and done well, God damn it, even if it meant spending hours hunched over a greasy gearbox, or painstakingly adjusting lights hot enough to toast a bagel.

I'll take praise for a difficult job well done before that for an easy job done quickly, any day of the week. I'd rather be creative than quick, diligent than gifted, and capable than "smart". "Smart" is bullshit.
posted by Commander Rachek at 4:59 PM on October 30, 2009 [4 favorites]


I'm your classic bad-at-math kind of person. I'm not scared of it, but I have little skill in it. I did well with geometry and statistics, two areas where there's a clear visualization or application for the math. Perhaps that has something to do with it. Algebra and calculus were simply abstract thought systems I did not understand the purpose or aims of, and so I perhaps rebelled against learning them. It seemed like a leap of faith, or rote memorization of rules and processes that appeared arbitrary.

However I never had anything against science classes. I did fine through college level chemistry and physics, both of which are quite technical. Again, it all related to the world in a way I could grasp. I know that algebra and calculus also relate to the world but I never developed an intuition for how.

Now. Since we're on the internet, I would like to point out that, as a student of English and writing, I have a lot more to complain about than you. Does grammar have big scary teeth or what?
posted by scarabic at 9:40 PM on October 30, 2009 [2 favorites]


not scared by math just simply suck at it. i always had problems in math even in like third grade. for some reason my brain is just simply not good with numbers, now science though it involes math i actually liked and diid real well in, i guess it just depends on the individual
posted by blessedangelnson at 1:47 AM on October 31, 2009


I think mathematicians have traditionally focussed almost exclusively on aesthetics grumblebee. Why else do so many math PhD pursue research in fields short on jobs when there is so much interesting mathematics that's extremely employable, both inside and outside academia. I feel the the biggest challenge for undergraduate math education is convincing smart kids that functional analysis, PDEs, and probability are just as interesting as all these great conjectures in number theory and set theory.
posted by jeffburdges at 5:33 AM on November 2, 2009


Mathematicians have focused on aesthetics because they are already converts. They have seen how math can be beautiful. The average person has no idea what it would even mean for math to be beautiful. I didn't get it for years, and only now I only fleetingly see it.
posted by grumblebee at 7:24 AM on November 2, 2009 [1 favorite]


Problem solving may be a lost skill not valued in education, but to really understand where the problem is, ask how it is possible that this nation can run so well-- all jokes aside--without it.

Grumblebee was right that most jobs don't require problem solving; and I'll confirm his suspicions about medicine. Furthermore, medicine (as an example) is moving further away from problem solving to a nearly total reliance on flowcharts and guidelines. Not there yet, but 9.8m/sec2.

So either a few talented problem solvers are keeping the entire nation running, maybe the world-- in which case we need to radically rethink social policy or prepare for mass unemployment as the robots take over these roles;

Or, problem solving simply isn't needed for day to day operations; and, like "leaders" they arise during a crisis;

Or the world has been coasting on the problem solvers of the past, and when that momentum inevitably slows... look out.
posted by TheLastPsychiatrist at 6:28 AM on November 7, 2009


If the decline in the day to day need for problem solving is real, I wonder if it has any relationship to the rising incidence of dementia. Use it or lose it?
posted by TheLastPsychiatrist at 6:35 AM on November 7, 2009 [1 favorite]


« Older Dallas Autism/Asperger evaluation?   |   Are Miller Indices used for anything, or are they... Newer »
This thread is closed to new comments.